Dermatology Final Question Collector PDF

You might also like

Download as pdf or txt
Download as pdf or txt
You are on page 1of 249

DERMATOLOGY

QUESTION COLLECTOR
Downloading, sharing and adding people to these docs is strictly forbidden.
This will not be tolerated and you shall be eliminated from all the docs, even if it’s Mohit.
FORMAT
● Font: CAIRO
● Size: 11
● Spacing: 1.5§
● Question in bold
● © = REPEAT QUESTION
● Please put some kind of photo in your profile picture

KEY
Answer = Ritika’s answer
Answer = Deepika's answer
Answer = INDER’s answer
Answer =Mohit’s answer
Answer =EMILIA’s answer
CONTENTS:

GROUP 3 TESTS 2019 3

DERMA CREDIT - GROUP 1,2 (KO) 3

SHORT TEST 2 - GROUP 1,2 (KO) 8

SHORT TEST 3 - GROUP 1,2 (KO) 9

SHORT TEST - GROUP 4 (BK) 10

ENTRANCE CREDIT 17

PPT QUESTIONS 23

VENEREOLOGY TESTS 26

CREDIT 2019 26

CREDIT 2018 30

RANDOM TEST 35

CREDIT 2010 39

RANDOM TEST 2 44

ENTRANCE CREDITS 50

2016/2018 CREDIT 50

2015/2016 ACNE CREDIT 56

RANDOM CREDITS 62

Random Credit 1 62

Random Credit 2 69

Random Credit 3 72

Random Credit 4 74

Random Credit 5 80

Random Credit 6 82

Random Credit 7 87

Random Credit 8 88
Random Credit 9 92

Random Credit 10 95

Random Credit 11 97

2018 FINAL 103

2017 CREDITS 124

2012 FINAL 171

2010 FINAL 173

2019 FINAL - POLISH DIVISION 202

2018 FINAL - POLISH DIVISION 223

BOOK QUESTIONS 237

GROUP 3 TESTS 2019

DERMA CREDIT - GROUP 1,2 (KO)


1. Pathogenesis of vitiligo is:
A. Neurogenic hypothesis
B. Self-destruct hypothesis
C. The autoimmune theory
D. All of the above

2. Koebner phenomenon doesn’t appear in


A. Psoriasis
B. Lichen planus
C. Warts
D. Impetigo
3. Patergy may occur in
A. Alopecia areata
B. Vitiligo
C. Pyoderma gangrenosum
D. Allergic vasculitis

4. The most common autoimmune blistering disease in adults:


A. Bullous pemphigoid
B. Pemphigus foliaceus
C. Pemphigus vulgaris
D. LABD

5. First line treatment of bullous pemphigoid is


A. topical clobetasol on the entire skin surface
B. topical clobetasol on skin lesions
C. topical hydrocortisone on skin lesions
D. B and C are correct

6. Major criteria for the diagnosis of atopic dermatitis ( Hanifin & Rajka) do not include:
A. pruritus
B. skin dryness
C. personal or family history of atopic diseases
D. chronic and relapsing course

7. Erythrasma is caused by
A. propionibacterium acne
B. Streptococcus pyogenes
C. trichophyton rubrum
D. corynebacterium minutissimum
8. Actinic keratosis:
A. can be treated with oral 5-FU
B. topical corticosteroid
C. photodynamic therapy
D. Topical Imiquimod
E. C&D are correct

9. Which of the following diseases does not typically involve mucosa


A. pemphigus vulgaris
B. Pemphigoid
C. lichen planus
D. candidiasis
E. keratoacanthoma

10. Clinical variants of BCC do not include


A. macular
B. nodular
C. morpheaform
D. ulcerative
E. superficial

11. At the early stage of borrelia burgdorferi infection may occur


A. erythema multiforme
B. erythema chronicum migrans
C. erythema nodosum
D. ecthyma
12. Breslow scale is used to describe
A. skin phototype
B. results of patch test
C. histopathologic assessed melanoma thickness
D. the level of skin sclerosis in systemic sclerosis

13. Indicate the possible treatment for mycosis fungoides from the options below
A. rituximab (anti CD20)
B. hydroxychloroquine
C. imiquimod
D. oral retinoids
E. topical retinoids

14. Which of the following is not related to chronic venous insufficiency


A. stasis dermatitis
B. atrophie Blanche
C. leg ulcers
D. lipodermatosclerosis
E. scleromyxedema

15. Which skin condition is related to sarcoidosis


A. lupus pernio
B. lupus vulgaris
C. lupus erythematosus
D. brownish purple infiltrated plaques
E. A & D are correct
16. The classification of psoriasis does not include
A. psoriasis vulgaris
B. psoriatic erythroderma
C. pustular psoriasis
D. ulcerative psoriasis

17. The pathogenesis of PLEVA is


A. bacterial
B. viral
C. autoimmune
D. unknown

18. In the third stage of rosacea we observe


A. telangiectasia
B. erythema
C. papules
D. nodules
E. all of the above

19. In differential diagnosis of Scabies we can consider


A. bullous pemphigoid
B. adverse cutaneous drug reaction
C. metabolic pruritus
D. atopic dermatitis
E. all of the above
SHORT TEST 2 - GROUP 1,2 (KO)
1. In MF (Mycosis Fungoides) treatment we use:
a. Prednisone
b. Methotrexat
c. Oral isotretinoin
d. PUVA
e. Both c and d

2. In Sezary syndrome we don't observe:


a. Diffuse hyperkeratosis
b. Red man syndrome
c. Diffuse hair loss
d. Single red patches or plaques with scaling
e. Peripheral lymphadenopathy

3. Select true (T) or false (F)


a. Alopecia areata (AA) is non scarring hair loss. TRUE/F
b. Middle age of onset in AA is 50. T/FALSE
c. AA can be treated with intralesional triamcinolone acetonide. TRUE/F
d. Spontaneous remission of AA is not possible. T/FALSE
e. AA universalis is a total loss of all terminal body and scalp hair. TRUE/F

4. First line treatment of bullous pemphigoid is:


a. topical clobetasol on the entire skin surface
b. topical clobetasol on skin lesions
C. topical hydrocortisone on skin lesions
D. b and c are correct

5. What kind of skin lesions is not characteristic to dermatomyositis.


a. Calcification
b. Gottron papules
c. Poikiloderma
d. Malar rash
e. Periorbital heliotrope flush

6. Select false answer about rosacea:


a. It occurs predominantly in females, but rhinophyma occurs mostly in males.
b. Typical lesions are erythema, telangiectasia and comedones.
c. Metophyma is a swelling of the forehead.
d. In topical treatment we use metronidazole gel or cream.

SHORT TEST 3 - GROUP 1,2 (KO)

1. In differential diagnosis of erythema nodosum we can consider


A. panniculitis
B. nodular vasculitis
C. lymphoma
D. all of the above
E. B & C

2. What is the fluorescent color in woods lamp in erythrasma infection?

A. green
B. coral red
C. white
D. no fluorescence

3. In stage 1 (localized infection) Lyme disease we can find skin manifestations except:
A. erythema migrans
B. diffuse urticaria
C. lymphocytoma
D. acrodermatitis chronica atrophicans

4. The classification of psoriasis does not include

A. psoriasis vulgaris
B. psoriatic erythroderma
C. pustular psoriasis
D. ulcerative psoriasis

5. Please write the 6 P's related to lichen planus

1. papules
2. polygonal
3. plaque
4. purple
5. pruritus
6. papules

SHORT TEST - GROUP 4 (BK)


1. Indicate a disease that can not be a cutaneous drug eruption:
a. Erythema multiforme
b. Lichen planus
c. Atopic dermatitis
d. TEN
e. Urticaria
2. Chronic urticaria is defined as urticaria with recurrent episodes lasting longer than:
a. 2 weeks
b. 6 weeks
C. 3 months
d. 6 months
E. 1 year

3. The skin of a person with atopic dermatitis is more susceptible to:


a. Warts
b. Herpes simplex infections
c. Staphylococcus aureus infections
d. Candida spp. infections
e. All of the above

4. The most common location of oral lichen planus is:


a. Buccal mucosa
b. gingiva
C. tongue
D. hard palate
E. soft palate

5. Erythroderma:
a. Is also known as erythrasma and is caused by Corynebacterium minutissimum
b. Is a long-term skin condition characterized by facial redness and papules
c. Is an acute streptococcal skin infection presented typically with a skin rash
D. is characterized by generalized erythema (over 90%)
e. Is one of the most common variants of psoriasis

6. CREST syndrome is characterized by:


a. Calcinosis
b. Erosive lesions of fingers
c. Sclerodactyly
D. a,b,c correct
E. a, c correct

7. Mark the true sentence about psoriasis:


a. Patients with nail psoriasis are predisposed to have joint involvement (psoriatic arthritis) more
often than those without nail changes
b. Intravenous corticosteroids are the best treatment option for psoriatic arthritis (Methotrexate)
c. Generalized pustular psoriasis does not affect children
d. Flexural psoriasis is associated with Auspitz sign (Acute guttate)
e. Psoriatic changes of the nail bed result in the "wax candle sign" (Chronic stable)

8. The typical psoriatic sign is:


a. Nikolsky sign
b. Apple jelly sign
c. Darier sign
d. Koebner phenomenon
e. Pathergy

9. The most common form of chronic cutaneous lupus erythematosus is:


a. Lupus tumidus
b. Chilblain lupus
c. Discoid lupus erythematosus
d. Lupus profundus
e. Malar lupus

10. Which antinuclear antibody is characteristic for renal involvement in systemic lupus
erythematosus?
a. Anti- histone
b. Anti — Ul- RNP
c. Anti- dsDNA
d. Anti- Sm
e. c and d

11. All of the below are clinical criteria of systemic lupus erythematosus with the EXCEPTION of:
a. High complement
b. Thrombocytopenia
c. Hemolytic anemia
d. Oral ulcers
e. Discoid lupus erythematosus lesions

12. What is the first line oral treatment for discoid lupus erythematosus?
a. Chloroquine
b. Belimumab
c. Acitretin
d. Cyclophosphamide
e. Etanercept

13. Indicate the treatment that is NOT used as topical treatment for psoriasis:
a. Topical crisaborole
b. Topical calcipotriol + betamethasone
c. Topical anthralin (cignoline)
d. Topical very potent steroid
e. Topical tar

14. Mark the drugs that are NOT used as systemic treatment for chronic plaque psoriasis:
a. Ustekinumab (anti-IL 12/23), Acitretin, Cyclosporin A
b. Rituximab (anti-IL 20), Cyclophosphamide
c. Methotrexate s.c., Apremilast (PDE4 inhibitor),
d. Infliximab (anti -TNFalpha), Secukinumab (anti IL-17)
e. All of the above can be used

15. Nikolsky sign is characteristic for:


a. Guttate psoriasis
b. Chronic plaque psoriasis
c. Inverted psoriasis
d. Palmo-plantar pustulosis
e. Generalized pustular psoriasis von Zumbusch type

16. Late onset psoriasis (Type II psoriasis)


a. Familial occurrence is easy recognizable
b. Guttate type lesions are characteristic for this variant (early onset)
c. Typical exacerbations usually occur after fungal infections
d. HLA Cw6 is typical
e. All answers are wrong

17. Systemic retinoids are first-line treatment for:


a. Scalp psoriasis
b. Nail psoriasis
c. Palmo-plantar pustulosis
d. Psoriatic arthritis
e. All answers are wrong

18. Mark common findings for DLE, SCLE and systemic SLE:
a. Presence of skin lesions
b. Hypersensitivity to UV light
c. Immunoglobulin deposits at the dermo-epidermal junction (lupus band)
d. a,b, c are correct
e. all answers are wrong

19. Skin findings in SLE do not include:


a. mucosal erosions
b. DLE lesions
c. Perioral dermatitis
d. Erythema of fingers, foot soles and nails folds
e. Diffuse alopecia
20. Morphea
a. Methotrexate may be used for the treatment
b. Is a synonym of limited systemic sclerosis
c. Raynaud phenomenon is usually present
d. Atrophy of alae nasi is typical for this disease
e. ACA (anti- centromere antibodies) are usually present

21. Treatment of systemic sclerosis includes:


a. Sildenafil
b. Nifedipine
c. Alprostadil
d. Methotrexate
e. All above can be used

22. Lichen planus


a. Reticular lesions on buccal mucosa predispose to cancer (squamous cell carcinoma)
b. Oral erosions are always present
c. Pruritus is present in minority of patients with skin involvement
d. Vulvo-vaginal form is usually erosive
e. May be associated with Helicobacter infection

23. ACR/EULAR criteria for the classification of systemic sclerosis do not include evaluation of:
a. Erythema
b. Skin hardening of the hands
c. Sclerodactyly
d. Erosive lesions/ulcerations on fingers
e. Telangiectasia

24. Dorsal pterygium is characteristic for nail involvement in the course of:
a. psoriasis
b. onychomycosis
c. lichen planus
d. pemphigus vulgaris
e. candidiasis

25. Subacute cutaneous lupus erythematosus may mimic:


a. psoriasis
b. lichen planus
C. erysipelas
D. acne
e. erythema nodosum

26. In the case of a heart-block in newborn child its mother should be screened for the presence:
a. lupus anticoagulant
b. circulating anti Dsg I antibodies
C. anti Dsg3 antibodies indirect immunofluorescence
d. dsDNA antibodies
e. the presence of Ro and La antibodies

27. The first line treatment of severe chronic spontaneous urticaria is:
a. cyclosporine A
b. plasmapheresis
c. second generation antihistamines
d. topical and systemic corticosteroids
e. omalizumab

28. The most common contact allergen in Europe and the USA is:
a. gluten
b. nickel
C. chromium
D. fragrances
E. latex
29. Not typical for dermatomyositis is :
a. Heliotrope
b. V-sign
c. Cicatricial alopecia
d. Gottron sign
e. Elevated liver enzymes (AST and ALT)

30. Collarette desquamation is characteristic for:


a. pityriasis rosea
b. dermatitis herpetiformis
C. pemphigus
d. atopic dermatitis
E. psoriasis

31. Patient with dermatomyositis should be screened for:


a. pulmonary hypertension
b. fungal infection
C. neoplasm
D. underlying neurological disease
E. diabetes

32. What is not typical for erythema nodosum:


a. lesions usually appear on anterior part of shins
b. nodules are usually ulcerated
C. tenderness lesions on palpation
D. disease can be provoked by Yersiniosis
E. drugs can be causative agents

ENTRANCE CREDIT
1. Which of the following lesion usually leads to scarring
A. Lichenification
B. Scaling
C. Wheal
D. Macule
E. Ulcer

2. Lesions that are not characteristic for acne:


A. Papules
B. Nodules
C. Pustules
D. Blisters
E. Cysts

3. Allergy to chromium is related to

A. Type I allergic mechanism

B. Type II allergic mechanism

C. Type III allergic mechanism

D. Type IV allergic mechanism

4. Pustules:

A. can be observed in some variants of psoriasis

B. can be seen in inflammatory acne

C. are typical lesions for folliculitis

D. all answers are correct


5. Hemidesmosomes are:

A. located between epithelial cells

B. connecting epithelial cells with basal membrane

C. are part of the endosome system of dermal phagocytes

D. are endosomes of sebocytes

6.Antinuclear antibodies examination is a routine procedure in the diagnosis of the following diseases,
EXCEPT:

A. systemic lupus erythematosus

B. mixed connective tissue disease

C. pemphigoid

D. systemic sclerosis/scleroderma

7. Breslow scale:

A. is used to grade melanoma thickness

B. is used to grade urticaria itching

C. is use to grade SLE photosensitivity

D. is used to grade inflammatory acne lesions

8. Viral diseases are following, EXCEPT:


A. Plane warts

B. Plantar warts

C. Common warts

D. Seborrheic warts

9. An example of autoimmune blistering disease is:

A. Dermatitis herpetiformis

B. SSSS

C. Morphea generalisata

D. Impetigo bullosa

10. High risk melanoma is related to:

A. Type IV skin phototype

B. Chronic sun exposure

C. Sunburns in childhood

D. HPV infection of melanocytes

11. The most important role in the protection against transepidermal water loss is played by:

A. Basal layer

B. Spinal layer
C. Granular layer

D. Cornified layer

12. Melanocytes are located:

A. between basal cells

B. between spinal cells

C. just below basal membrane zone

D. just below granular layer

13. Mast cells are:

A. residual in the epidermis

B. playing major role in type II allergic reactions

C. are equipped with high number of high affinity IgE receptor (FceRI)

D. playing the major role in pathogenesis autoimmune connective tissue diseases

14. Wheal is characteristic for

A. atopic dermatitis

B. urticaria

C. guttate psoriasis

D. erythrasma
15. Pemphigus vulgaris

A. is a mild disease of pilosebaceous unit

B. is a bacterial disease

C. typically pustules are present

D. all of the above are correct

E. all of the above are wrong

16. Systemic isotretinoin

A. is used in the treatment of dermatomyositis (steroids)

B. is highly teratogenic

C. is used for the treatment of melanoma

D. is never used as systemic agent

17. The true statement of atopic dermatitis is:

A. The percentage of affected people has decreased in the last 40 years

B. It affects higher rate of population in developing countries when compared to western countries

C. The prevalence of atopic dermatitis is higher in children than in adults

D. It rarely affects asthmatic patients


18. Indicate viral diseases

A. plane warts

B. seborrheic wart

C. Zoster

D. Bowen’s disease

19. Indicate bacterial diseases

A. Erysipelas

B. Erythrasma

C. Pityriasis versicolor

D. molluscom contagiusum

20. For the treatment of acne one should not use:

A. Cyclosporine A

B. Oral antibiotics

C. Topical antibiotics

D. Topical retinoids

PPT QUESTIONS

1. You are called by the mother of a four-year-old patient. The mother reports her child has a
painful sunburn involving the face, shoulders and back. She denies the presence of blistering
of the skin. The following is appropriate advice to provide:
A. administer an oral anti-inflammatory such as ibuprofen
B. apply a topical pain reliever such as benzocaine
C. avoid applying moisturising lotions, creams or ointments to the skin during this time
D. initiate use of hydrocortisone 2.5% ointment if blisters develop
E. put child in an ice bath to cool down the hot burning skin

2. An 18 year old white female returns to your clinic for acne. She has comedones that have improved
with topical retinoid therapy and benzoyl peroxide, but she still gets pustules and inflammatory
papules. She plays field hockey. You decide to prescribe doxycycline 100 mg twice-daily. Which of the
following side effects do you need to warn her about?

A. Decreases efficacy of birth control pills


B. hyperpigmentation
C. hypertriglyceridemia
D. photosensitivity
E. staining of the teeth

3. A 68 year old Vietnam veteran comes to your clinic for evaluation of high blood pressure and high
cholesterol. He asks you in passing about some dryness and Mild itching on both his feet for several
years that has not improved with 1% hydrocortisone cream and moisturizers. What is the next step in
diagnosis?

A. Patch testing for allergic contact dermatitis


B. potassium hydroxide exam in clinic
C. punch biopsy to rule out squamous cell carcinoma in situ or mycosis fungoides
D. serum protein electrophoresis to search for immunosuppression
E. titers for agent orange exposure

4. A 26 year old male presents with 2 years of white spots on his neck, upper chest, and back, that are
more noticeable in the spring and summer months and get better in winter. You perform a KOH exam
in the office as shown. What treatment would you recommend?

A. Desonide cream
B. pimecrolimus cream
C. oral prednisone
D. Selenium sulfide lotion
E. UV light therapy

5. Mrs. H, is a 30 year old female who presents with several months of worsening providing dry scaly
skin between the toes and over the lateral aspect of the foot. More recently she developed blisters
on the plantar aspect of the foot beneath the toes. She does not report any new shoes or lotions to
the feet. Which diagnosis is MOST likely?

A. Contact dermatitis
B. dyshidrotic eczema
C. friction blisters
D. psoriasis
E. tinea pedis

6. A 4 month old girl presents with beefy red plates in the groin area with satellite pustules for one
week. She cries with diaper changes and has not improved with good Barrier cream such as zinc oxide
paste. What is the best Next Step?

A. Prescribe a topical antifungal agent such as nystatin or an imidazole


B. prescribe an oral antibiotic
C. prescribe a combination therapy with high potency topical steroid plus an antifungal
agent
D. prescribe a topical antibacterial agent such as mupirocin
E. prescribe a potent topical corticosteroid

7. A 52 year old healthy male complains of 10 years of thickening of the great toe nails with crusty
debris, and he's worried it's spreading to other toes. Which of the following do you recommend first?

A. Ciclopirox nail lacquer twice daily for 6 months


B. empirically begin oral fluconazole 150 mg weekly for 6 months
C. empirically begin oral Ketoconazole 400 mg daily for 3 months
D. empirically begin oral terbinafine 250 mg daily for 3 months
E. perform fungal culture or KOH to confirm diagnosis

VENEREOLOGY TESTS

CREDIT 2019
The marked answers are from the teachers key

1. Lymph nodes in case of syphilis


A. are small and painful
B. have a tendency to soften up and great fistulas
C. their increase in size has always a generalized character
D. no not increase in size
E. are enlarged, hard and painless

2. Choose the incorrect relation


A. gonorrhoeae - epididymitis
B. chlamydiosis - cystitis
C. syphilis - alopecia
D. chlamydiosis - Reiter’s syndrome
E. genital warts - recurrence of lesions

3. Vulvar intraepithelial neoplasia (VIN) is induced by:


A. HPV 1 and HPV 3
B. HPV 6 and HPV 11
C. HPV 1 and HPV 6
D. HPV 16 and HPV 18
E. HPV 6 and HPV 184
4. Choose the INCORRECT information about the course of genital herpes infection in patients infected
with HIV
A. relapses more often
B. symptoms during relapses last longer
C. symptoms during relapses are less intense
D. symptoms during relapses cover a bigger surfaces
E. patients often need a chronic drug treatment

5. Woman with secondary syphilis (lues secundaria recidivans)


A. is very infectious towards her sexual partner and very infectious towards the foetus
B. is very infectious toward her sexual partner and not infectious towards the foetus
C. is not infectious either toward sexual partner nor towards her foetus
D. is not infectious towards the sexual partner but infectious toward the foetus
E. none of the above is correct

6. Serology blood test is one of the basic diagnostic tests in


A. syphilis
B. syphilis, HIV
C. syphilis, HIV, chlamydiosis
D. syphilis, HIV, chlamydiosis, gonorrhoeae
E. syphilis, HIV, chlamydiosis, gonorrhoeae, herpes

7. Which of the following is NOT a symptom of early acquired syphilis


A. eyebrows and eyelashes alopecia
B. increased number of leukocytes in the blood
C. high ESR (erythrocyte sedimentation rate)
D. macular, papular, pustular and vesicular eruptions
E. regional or generalized lymphadenopathy

8. Changes in the CNS caused by Treponema Pallidum infection are observed only in patients with late
syphilis:
TRUE
FALSE

9. Condylomata lata (hypertrophic syphilic papules) due to their size and lack of remission during
routinely used therapy requires removal by laserotherapy, cryotherapy or surgically.
TRUE
FALSE

10. Choose the correct sentence about gonorrhoeae in male patients


A. discharge from the urethra is abundant and purulent
B. symptoms appear after 3-5-7 weeks
C. symptoms of dysuria occur sporadically
D. proctitis is never observed
E. treatment with procaine penicillin lasts at least 20 days

11. How do coexistent symptoms of sexually transmitted diseases influence the risk of HIV infection
during sexual contact?
A. has no influence
B. increases the risk
C. decreases the risk

12. Which of the following is NOT a characteristic symptom of primary syphilis


A. pain
B. hard base infiltration
C. self-healing after 2-3 weeks
D. appearance 3.4 weeks after infection
E. regional lymphadenopathy

13. Kaposi sarcoma in a person infected with HIV can evolve


A. on the mucous membrane
B. in the internal organs
C. in the lymph nodes
D. on the skin
E. all of the above

14. Oedema induratum


A. concerns only male
B. is usually a complication of secondary syphilis
C. is painful
D. is connected with the lymph stasis
E. concerns only women

15. In the tabes dorsalis we can observe


A. deep sensation disturbances
B. decrease tendon reflexes
C. penetrating ulcer of the feet
D. Argyll Robertson syndrome
E. all of the above are correct

16. Choose the correct statement regarding Hoigne syndrome


A. it can appear during 48 hours after receiving drugs
B. auditory hallucinations are not apparent
C. heart rate is decreased
D. blood pressure is increased
E. patient is not excited

17. Anogenital infections caused by yeasts are more frequently observed in


A. patients with diabetes mellitus
B. HIV infected patients
C. patients undergoing antibiotic therapy
D. immunocompromised patients
E. all of the above are correct

18. Which test is nontreponemal


A. TPHA
B. TP-PA
C. RPR
D. FTA
E. capita syphilis IgM

19. Non-gonococcal urethritis most frequently is caused by:


A. Ureaplasma urealyticum
B. Chlamydia trachomatis
C. Gardnerella vaginalis
D. Trichomonas vaginalis
E. candida albicans

20. The shortest time of incubation is in


A. syphilis
B. gonorrhoeae
C. chlamydiosis
D. trichomoniasis
E. HPV infection

CREDIT 2018
1. In congenital syphilis treatment decisions frequently must be made on the basis of:
A. identification of syphilis in the mother
B. adequacy of maternal treatment
C. presence of clinical, laboratory, or radiographic evidence of syphilis in the infant
D. comparison of maternal (at delivery) and infant
E. all the above parameters

2. The Jarisch-Herxheimer reaction is:


A. acute febrile reaction frequently accompanied by headache, myalgia
B. usually occur within the first 24 hours after any therapy of syphilis
C. occurs most frequently among patients who have early syphilis
D. antipyretics may be used, but they have not been proven to prevent this reaction
E. all answers are correct

3. Category C of clinical symptoms in HIV positive patient includes


A. persistent lymphadenopathy
B. hairy leukoplakia
C. macular rash
D. psoriatic lesion
E. invasive cervical cancer

4. The longest time of incubation (i.e time to appearance of first clinical symptoms) is in:
A. syphilis
B. chlamydiosis
C. gonorrhea
D. trichomoniasis
E. HPV infection

5. Papular rash in secondary syphilis may mimic


A. varicella
B. rosacea
C. contact dermatitis
D. psoriasis
E. urticaria

6. Primary lesion in early syphilis


A. is painless
B. usually develops within 6-8 weeks after infection
C. usually presents as disseminated macular rash
D. heals with scar formation
E. never develops in extragenital areas

7. Genital ulcer diseases (GUD) include:


A. genital herpes
B. syphilis
C. chancroid
D. all of the above
E. none of them

8. Treatment of chlamydia trachomatis infections does not include


A. doxycycline
B. erythromycin
C. metronidazole
D. amoxicillin
E. azithromycin

9. The most common STI is caused by:


A. chlamydia
B. HSV
C. HBV
D. HPV
E. HIV

10. Vulvar intraepithelial neoplasia (VIN) is induced mainly by:


A. HPV1 and HPV3
B. HPV 6 and HPV 11
C. HPV1 and HPV 6
D. HPV 16 and HPV 18
E. HPV 6 and HPV 18
11. In contrast to adult secondary syphilis, congenital syphilis may have:
A. papular lesions
B. lymphadenopathy
C. vesicular lesion
D. positive serological tests
E. hepatitis

12. Complication of the primary lesion in syphilis do not include:


A. scar formation
B. phimosis
C. paraphimosis
D. oedema induratum
E. secondary infections

13. Which test is nontreponemal:


A. TPHA
B. TP-PA
C. RPR
D. FTA
E. capita IgM

14. Alopecia syphilitica


A. lasts for about 1 month
B. is always associated with cutaneous lesions
C. do not regress spontaneously
D. all correct
E. all incorrect

15. Tertiary syphilis refers to


A. cardiovascular syphilis only
B. gummata and neurosyphilis
C. cardiovascular syphilis and gummata
D. neurosyphilis and cardiovascular syphilis
E. neurosyphilis only

16. Patients with acute retroviral infection may have


A. fever
B. lymphadenopathy
C. maculopapular rash
D. neurological abnormalities
E. all answers correct

17. Symptoms of gonorrhea in women usually do not include


A. cervicitis
B. urethritis
C. arthritis
D. proctitis
E. oropharyngitis

18. What treatment is not effective in condylomata acuminate


A. condyline
B. imiquimod
C. penicillin
D. cryotherapy
E. laser therapy

19. Non-gonorrheal urethritis (NGU) most frequently is caused by:


A. ureaplasma urealyticum
B. chlamydia trachomatis
C. Gardenella vaginalis
D. Trichomonas vaginalis
E. Candida albicans
20. The earliest diagnosis of acquired syphilis can be established by
A. dark field microscopy
B. FTA
C. PRP
D. TP-PA (TPHA)
E. VDRL

RANDOM TEST
1. Genital ulcers caused by
A. herpes
B. chancre
C. all of them
D. none of them

2. The longest incubation period


A. herpes
B. gonorrhea
C. HPV
D. HBV
E. HIV

3. Non-treponemal screening test


A. TPA
B. TPHA
C. capita IgM antibody
D. RPR

4. Category C HIV symptoms


A. Lymphadenopathy
B. leukoplakia
C. invasive CIN

5. Which one is not effective for treatment of condylomata acuminata?


A. podophyllin
B. penicillin
C. cryotherapy
D. laser therapy

6. Chlamydia trachomatis infection cannot be treated with


A. metronidazole
B. erythromycin
C. azithromycin
D. ampicillin
E. doxycycline

7. Papular rash in secondary syphilis may mimic


A. varicella
B. psoriasis
C. contact dermatitis
D. rosacea
E. urticaria

8. Most common STI:


A. Chlamydia
B. gonorrhea (second most common)
C. HPV
D. HIV
E. HIV

9. Alopecia syphilitica
A. at least 1 month
B. Chennai super kings
C. do not regress spontaneously
D. antibody negative
E. all are incorrect

10. Earliest diagnosis method for syphilis


A. RPR
B. VDRL
C. dark field microscopy
D. TPA

11. Difference primary lesion between congenital syphilis and acquired syphilis
a) papules
b) macule
c) vesicle
d) erosion

12. Genital warts are induced mainly by


a) HPV1 and HPV3
b) HPV 6 and HPV 11
c) HPV1 and HPV6
d) HPV6 and HPV16
e) HPV16 and HPV18

13. Patients with acute retroviral infection may have


a) fever
b) lymphadenopathy
c) maculopapular rash
d) neurological abnormalities
e) all answers are correct

14. Primary lesion in early syphilis


a) develops within 6-8 weeks of infection
b) usually presents as disseminated maculopapular rash
c) heals with scar formation
d) is painless
e) never develops in …..area

15. Complications of primary syphilis do not include:


a) phimosis
b) paraphimosis
c) oedema induratum
d) secondary infection
e) scar formation

16. Tertiary syphilis refers to


a) cardiovascular syphilis and gummata
b) cardiovascular syphilis only
c) neurosyphilis and cardiovascular syphilis
d) neurosyphilis only
e) gummata and neurosyphilis
f) all correct
17. In congenital syphilis treatment decisions frequently must be made on the basis of
a) identification of syphilis in the mother
b) adequacy of maternal treatment
c) presence of clinical, laboratory, or radiographic evidence of syphilis in the infant
d) comparison of maternal (after delivery) and infant nontreponemal serologic tests by using the same
test and preferably the same laboratory
e) all the above parameters

18. Symptoms of gonorrhoea in women usually do not include


a) cervicitis
b) urethritis
c) proctitis
d) oropharyngitis
e) arthritis (Is a symptom IF DISSEMINATED)

19. Non-gonorrhoeal urethritis is most frequently caused by


a) Ureaplasma urealytica
b) Gardenella vaginalis
c) Trichomonas vaginalis
d) Chlamydia trachomatis
e) Candida albicans

20. Jarisch-Herxheimer syndrome


a) appear 7-8 hrs after treatment
b) fever and myalgia
c) antipyretic won't help
d) have nothing to do with Chennai super kings
e) all above are true

CREDIT 2010
1. The shortest time of incubation:
(a) syphilis
(b) Chlamydia
(c) gonorrhoea
(d) HIV infection
(e) HPV infection

2. The most common STI is caused by


(a) Chlamydia
(b) HSV
(c) HPV
(d) HBV
(e) HIV

3. Which positive test will prove congenital syphilis:


(a) VDRL
(b) TP-PA (TPHA)
(c) FTA
(d) Capita IgM
(e) RPR

4. What treatment is not effective in genital warts (condylomata accuminata)


(a) condyline
(b) acyclovir
(c) imiquimod
(d) cryotherapy
(e) laser therapy

5. The earliest diagnosis of acquired syphilis can be established by


(a) FTA
(b) PRP
(c) TP-PA (TPHA)
(d) Dark field microscopy
(e) VDRL

6. Category B of clinical symptoms in HIV positive patient includes


(a) persistent lymphadenopathy
(b) long-lasting genital herpes
(c) hairy leukoplakia
(d) cutaneous T cell lymphoma
(e) cutaneous B cell lymphoma

7. Genital warts are induced mainly by


(a) HPV1 and HPV3
(b) HPV6 and HPV11
(c) HPV1 and HPV6
(d) HPV6 and HPV16
(e) HPV16 and HPV18

8. Patients with acute retroviral infection may have


(a) fever
(b) lymphadenopathy
(c) maculopapular rash
(d) neurological abnormalities
(e) all answers are correct

9. Papular rash in secondary syphilis may mimic


(a) varicella
(b) lichen planus
(c) contact dermatitis
(d) rosacea
(e) urticaria

10. Primary lesion in early syphilis


(a) develops within 6-8 weeks of infection
(b) usually presents as disseminated maculopapular rash
(c) heals with scar formation
(d) is painless
(e) never develops in …..area

11. Complications of primary syphilis do not include:


(a) phimosis
(b) paraphimosis
(c) oedema induratum
(d) secondary infection
(e) lichenification

12. Treatment of condylomata lata includes


(a) cryotherapy
(b) laser therapy
(c) penicillin
(d) imiquimod
(e) acyclovir
(f) all correct

13. In contrast to adult secondary syphilis, congenital syphilis may have


(a) popular lesions
(b) vesicular and bullous lesions
(c) lymphadenopathy
(d) positive serological tests
(e) hepatitis

14. Alopecia syphilitica


(a) usually develops 2 years after infection
(b) lasts about 1-2 months
(c) sometimes may be the only one clinical sign of syphilis
(d) does not regress spontaneously
(e) serologic tests are negative

15. Tertiary syphilis refers to


(a) cardiovascular syphilis and gummata
(b) cardiovascular syphilis only
(c) neurosyphilis and cardiovascular syphilis
(d) neurosyphilis only
(e) gummata and neurosyphilis
(f) all correct

16. Genital ulcer disease (GUD) [causes] do not include


(a) genital herpes
(b) gonorrhoea
(c) syphilis
(d) chancroid
(e) all above

17. In congenital syphilis treatment decisions frequently must be made on the basis of
(a) identification of syphilis in the mother
(b) adequacy of maternal treatment
(c) presence of clinical, laboratory, or radiographic evidence of syphilis in the infant
(d) comparison of maternal (after delivery) and infant nontreponemal serologic tests by using the same
test and preferably the same laboratory
(e) all the above parameters

18. Symptoms of gonorrhoea in women usually do not include


(a) cervicitis
(b) urethritis
(c) proctitis
(d) oropharyngitis
(e) conjunctivitis
19. Non-gonorrhoeal urethritis is most frequently caused by
(a) Ureaplasma urealytica
(b) Gardnerella vaginalis
(c) Trichomonas vaginalis
(d) Chlamydia trachomatis
(e) Candida albicans

20. Treatment of Trichomonas includes


(a) metronidazole
(b) doxycycline
(c) erythromycin
(d) amoxicillin
(e) procaine penicillin
(f) all correct

RANDOM TEST 2
From question 10 onwards, use this key. Otherwise, normal mcqs.
A: all true or false
B: 1,2,3 true
C: 1,2,4 true
D: 1,3,4 true
E: 2,3,4 true

1. What treatment is effective in keratitis parenchymatosa:


A. Procaine penicillin
B. Crystalline penicillin
C. Tetracyclines
D. Macrolides
E. Steroid
2. Which positive test will prove syphilis in a newborn:

1. VDRL
2. TPHA
3. FTA
4. Captia IgM-
5. RPR

3. The risk of infecting fetus by HIV women given retroviral therapy is:

1. 100%
2. 50%
3. 30%
4. 2%
5. None

4. Meningovascular syphilis and cardiovascular syphilis do not have in common:

1. Endarteritis obliterans
2. Belong to late syphilis
3. Belong to secondary syphilis
4. Good response to early penicillin treatment

5. Which test is quantitative and not cardiolipin:

1. RPR
2. USR
3. VDRL
4. FTA
5. TPHA

6. Serology is not useful in:

1. Latent early syphilis


2. Latent late syphilis
3. Concomitant syphilis and HIV
4. Chlamydia trachomatis inf.
5. Congenital syphilis

7. The shortest incubation time is in:

1. Syphilis
2. Chlamydiosis
3. Gonorrhea
4. HIV
5. HPV

8. The highest risk of infection after one sexual contact:

1. Syphilis
2. HIV
3. Chlamydiosis
4. HPV
5. HSV

9. Acquired syphilis and congenital syphilis do not have in common:


1. Papular rash
2. Condylomata lata
3. Bullous rash
4. …...

10. Treponema pallidum penetrates: ALL TRUE OR FALSE

1. Abraded skin
2. Abraded mucosae
3. Intact mucosae
4. All true

11. Lymph nodes in primary syphilis:

1. Are firm to palpation


2. Are freely mobile
3. May soften (Never soften)
4. Are painless

ANSWER = C:1,2,4

12. Constitutional symptom of secondary syphilis may consist:

1. Low grade fever


2. Headache
3. Sore throat
4. Photophobia

ANSWER = All True


13. Phimosis may be a complication of:

1. Yeast infection
2. Contact dermatitis
3. Primary syphilis
4. Syndroma Stevens-Johnson
ANSWER = All True

14. Papular rash in secondary syphilis may mimic:

1. Psoriasis
2. Lichen planus
3. Pemphigoid
4. Impetigo
ANSWER = B(1,2,4)

15. Typical of secondary syphilis is:

1. Generalized rash
2. Resolving of eruptions without treatment
3. Generalized lymphadenopathy
4. Good response to penicillin treatment
ANSWER = All True

16. Congenital syphilis and acquired syphilis have in common:

1. Primary sore
2. Papular rash
3. Sore throat
4. Condylomata lata

ANSWER = E (2,3,4)
17. What is true in early syphilis:

1. Are infectious to sexual partner


2. May give birth to a baby with congenital syphilis
3. Serological tests for syphilis may be negative
4. Tendon reflexes may be exaggerated

ANSWER = B (1,2,3)

18. Patients with acute retroviral infection may have:

1. Fever
2. Generalized lymphadenopathy
3. Maculopapular rash
4. CD4 less than 400--

ANSWER = B (1,2,3)

19. Proliferation of condylomata acuminata may be encouraged by: ALL TRUE

1. Gonococcal inf.
2. HIV inf
3. Chlamydial inf
4. Steroid therapy

20. In pts. with HSV-2, you may observe: A or E (tror E)

1. Low titers of tests for syphilis


2. Flu like symptoms.
3. Risk of autoinoculation of cornea
4. Good response to ... steroids

ENTRANCE CREDITS

2016/2018 CREDIT

1. Which of the following lesion usually leads to scarring


F. Lichenification
G. Scaling
H. Wheal
I. Macule
J. none of the above

2. Acantosis nigricans
A. Is a paraneoplastic disease
B. Is a fungal infection caused by Dermatophytes nigricans
C. Is a bacterial infection caused by Corynebacterium nigricans
D. Is variant of solar keratosis
E. None of the above

3. Allergy to chromium is related to

A. Type I allergic mechanism

B. Type II allergic mechanism

C. Type III allergic mechanism

D. Type IV allergic mechanism


4. Pustules:

A. can be observed in some variants of psoriasis

B. can be seen in inflammatory acne

C. are typical lesions for folliculitis

D. all answers are correct

5. Hemidesmosomes are:

A. located between epithelial cells

B. connecting epithelial cells with basal membrane

C. are part of the endosome system of dermal phagocytes

D. are endosomes of sebocytes

6 . Antinuclear antibodies examination is routine procedure in the diagnosis of following diseases,


EXCEPT:

A. systemic lupus erythematosus

B. mixed connective tissue disease

C. pemphigoid

D. systemic sclerosis/scleroderma

7. Paraneoplastic diseases are the following, EXCEPT:


A. Erythema multiforme

B. Erythema gyratum repens

C. Necrolytic migratory erythema

D. Erythema annulare centrifugum

8. Viral diseases are following, EXCEPT:

A. Plane warts

B. Plantar warts

C. Common warts

D. Seborrhoeic warts

9. An example of an autoimmune blistering disease is:

A. Dermatitis herpetiformis

B. SSSS

C. Morphea generalisata a

D. Impetigo bullosa

10. High risk melanoma is related to:

A. Type IV skin phototype

B. Chronic sun exposure


C. Sunburns in childhood

D. HPV infection of melanocytes

11. The most important role in the protection against transepidermal water loss is played by:

A. Basal layer

B. Spinal layer

C. Granular layer

D. Cornified layer

12. Melanocytes are located:

A. between basal cells

B. between spinal cells

C. just below basal membrane zone

D. just below granular layer

13. Mast cells are:

A. residual in the epidermis

B. playing major role in type II allergic reactions

C. are equipped with high number of high affinity IgE receptor (FceRI)

D. playing the major role in pathogenesis autoimmune connective tissue diseases


14. Wheal is characteristic for

A. atopic dermatitis

B. urticaria

C. guttate psoriasis

D. erythrasma

15. Pemphigus vulgaris

A. is a mild disease of pilosebaceous unit

B. is a bacterial disease

C. typically pustules are present

D. all of the above are correct

E. all of the above are wrong

16. Systemic isotretinoin

A. is used in the treatment of dermatomyositis

B. is highly teratogenic

C. is used for the treatment of melanoma

D. is never used as systemic agent


17. The TRUE statement of atopic dermatitis is:

A. The percentage of affected people has decreased in the last 40 years

B. It affects higher rate of population in developing countries when compared to western countries

C. The prevalence of atopic dermatitis is higher in children than in adults

D. It rarely affects asthmatic patients

18. Which of the following are viral diseases ?

A. plane warts

B. seborrhoeic wart

C. Zoster

D. Bowen’s disease

19. Indicate bacterial diseases

A. Erysipelas

B. Erythrasma

C. Pityriasis versicolor

D. molluscum contagiosum

20. For the treatment of acne one should NOT use:

A. Cyclosporine A
B. Oral antibiotics

C. Topical antibiotics

D. Topical retinoids

2015/2016 ACNE CREDIT


1. An 18 year old white female returns to your clinic for acne. She has comedones that have
improved with topical retinoid therapy and benzoyl peroxide, but she still gets pustules and
inflammatory papules. She plays field hockey. You decide to prescribe doxycycline 100 mg 2X
daily. Which of the side effects do you need to warn her about.

A. Decreased efficacy of birth control pills


B. Hyperpigmentation
C. Hypertriglyceridemia
D. Photosensitivity
E. Staining of the teeth

2. A 15 year old female presents to clinic with acne with 30-40 comedones on the forehead cheeks
and chin with very few erythematous papules, and no scarring. She reports that topical benzoyl
peroxide isn’t working for her despite using it according to instruction for 6 months. She has no
involvement of chest or back and has normal menstrual periods. Which of the following is the best
addition to her acne treatment.

A. Oral Isotretinoin
B. Oral Minocycline
C. Oral Spironolactone
D. Topical Antibiotic
E. Topical Retinoid
3. Oral tetracyclines aren’t used in children under the age of 8 because:

A. Risk of bleaching of the hair


B. Risk of damage to tooth enamel and developing bones
C. Risk of hyperkalemia
D. Risk of hyperpigmentation of the skin

4. When prescribing medications for acne vulgaris,an important step is to establish reasonable
expectations with the patient. How long might the patient have to wait before seeing improvements
in his or her face.

A. 1-2 weeks
B. 3-4 weeks
C. 2-3 months
D. 4-6 months

5. Acne rosacea can be distinguished from acne vulgaris by which of the following features?

A. Absences of comedones
B. Distribution limited to the face (Individuals can have acne vulgaris limited to the face)
C. Inflammatory papules and pustules (Seen in both acne rosacea and acne vulgaris)
D. Irritation from topical products (true in both acne rosacea and acne vulgaris)

6. Which of the following medications is indicated in the treatment of acne in a pregnant woman?

A. Oral Doxycycline
B. Oral Isotretinoin
C. Oral Minocycline
D. Topical Clindamycin

7. Which of the following patients would be most appropriate to refer to a dermatologist?


A. Patient has both comedonal acne and few inflammatory lesions on the face, chest and back
B. Patient has no improvement in moderate acne after one month of treatment with topical
retinoid
C. Patient is a post adolescent female with new onset acne
D. Patient has extensive nodular lesions with cysts and diffuse scarring.

8. Select the factors that cause acne vulgaris:


A. Skin Keratosis disorder
B. Vasomotor disturbances
C. Helicobacter pylori
D. Herpes simplex virus

9. Skin lesion in a size of pea, increased consistency with or without epidermal hyperplasia is:
A. Cyst
B. Pimple
C. Pellet
D. Follicle

10. Skin disorder in secretion of melanin pigment (skin, hair, iris) is:
A. Melasma
B. Albinism
C. Dermatosis
D. porfiria

11. Mouth mucosa candidiasis is differentiated with:


A. Lichen planus
B. Leukoplakia
C. Aphthous
D. All of the above

12. Choose the WRONG characteristics of rosacea (WRITTEN NOTE ANSWERS: A,C,D)
A. Blackheads
B. Seborrheic skin
C. In time of puberty
D. Men more often than women

13. Choose the WRONG characteristics of Scabies


A. Is caused by scabies
B. Contagious
C. Lichenification may occur
D. Changes often occur in interscapular area

14. Actinic Keratosis:


A. Applies mostly to the elderly ones
B. Is neoplastic state
C. Applies to mostly sun-exposed areas
D. All the above are correct

15. Melanoma develops from:


A. Healthy skin
B. Congenital pigmented nevus
C. Atypical nevus pigmentosus
D. All of the above

2016 CREDIT
1. Subacute cutaneous lupus erythematosus may mimic:

A. Psoriasis
B. Erysipelas
C. Acne
D. Erythema nodosum
2. Psoriasis rarely affects:
A. Elbows
B. Knees
C. Scalp
D. Face

3. The first line treatment of severe idiopathic chronic urticaria is:

A. Cyclosporine A
B. Plasmapheresis
C. Second generation antihistamines
D. Topical and systemic corticosteroids

4. Calcineurin inhibitors are used for the treatment of :


A. Atopic dermatitis as topicals
B. Psoriasis as systemic drugs
C. Urticaria as systemic drugs
D. Urticaria as topicals

5. LICHEN PLANUS - which is NOT true:


A. Maybe associated with viral hepatitis
B. Typical localization is face
C. Wickham striae can be present on larger lesions
D. Can cause cicatricial Alopecia

6. Which systemic therapies should NOT be used in skin psoriasis:


A. Methotrexate
B. Cyclosporine
C. Corticosteroids
D. biological
7. Chief mediator in Urticaria is:
A. Acetylcholine
B. Histamine
C. Dopamine
D. chemokines

8. Mycosis fungoides is:


A. Th2- cell lymphoma
B. Distinguishes 3 periods
C. The course is chronic
D. All of the above

9. The duration of acute urticaria:


A. 6 weeks
B. Over 6 weeks
C. Upto 48 hours
D. Above 72 h

10. Most common contact allergens are:


A. Benzocaine, balsam of Peru
B. Neomycin, lanolin
C. Detreomycyna, cobalt
D. Chromium, nickel

11. Whan ofter harms aspirin indued urticarial?


A. Canned food
B. Eggs
C. Some fruits
D. milk
RANDOM CREDITS

Random Credit 1

1. What is the primary lesion in psoriasis?

A. Papule

B. Pustule

C. Plaque

D. Scale

2. What is true about psoriasis?

A. The Auspitz sign is characteristic for psoriasis

B. Koebner phenomenon occurs only in psoriasis

C. We use systemic steroids in generalized pustular psoriasis (topical)

D. In histopathology hyperkeratosis without parakeratosis is typical

3. Lichen planus:

A. could be associated with viral infections (for example HCV)

B. It’s one of the causes for non-cicatricial alopecia

C. systemic steroids are contraindicated in these condition

D. the lesions are not itchy


4. The most often cause of erythroderma in adults is:

A. seborrheic dermatitis

B. psoriasis

C. atopic dermatitis

D.skin lymphoma

5. Typical symptoms in systemic sclerosis are:

1. Microstomia

2. Sclerodactyly

3. Proteinuria

4. Hypersensitivity for light

A. 1,3

B. 1,2,3

C. 1,2

D. 1,2,3,4

6. What are you going to recommend in the treatment for patient with plaque psoriasis with joints
involvement, hypercholesterolemia and diabetes?

A. cyclophosphamide
B. acitretin

C. methotrexate

D. phototherapy

7. What is false about Erysipelas?

A. It’s contagious (it is possible to catch the disease from other family member)

B. the most typical localization are shins and face

C. in the treatment we use amoxicillin

D. the onset is acute with high fever and chills

8. Choose the correct answer:

A. Erythrasma is a fungal skin infection (bacterial)

B. Herpes zoster is as contagious as Varicella

C. Systemic Lupus erythematosus occurs most often in males

D. The most often localization for herpes zoster in the face

9. Choose the correct answer:

A. In TEN we have over 50% skin detachment (over 30%)

B. In SJS we have under 10% skin detachment

C. Systemic steroids are the first choice treatment in TEN (I.V immunoglobulins)
D. most often TEN is caused by bacterial infections (viral - HSV)

10. Acute urticaria:

A. even only one episode of acute urticaria requires performing allergy tests

B. Angioedema resolves within 72 hours

C. the first choice treatment are systemic steroids

D. acute urticaria should resolve within 12 weeks

11. Which treatment is NOT useful in urticaria:

A. Cetirizine

B. Topical glucocorticosteroids

C. Cyclosporine

D. Methylprednisolone

12. Which one of the answers below are not included in the Hanifin and Rajka criteria:

A. pruritis

B. Dermatitis affecting flexural surfaces in adults and the face and extensors in infants

C. Chronic and recurrent course

D. Red dermographism
13. Choose the correct answer about lichen planus:

1. Lesions can appear on the skin or inside the mouth

2. The primary lesion in pustule (papule)

3. Can cause cicatricial alopecia

4. Long lasting erosive lichen planus can result in squamous cell carcinoma

A. 1,2,3,4

B. 1,3,4

C. 1,3

D. 1,2,3

14. In the SLE criteria we don’t have:

A. Raynaud’s phenomenon

B. Arthritis

C. Serositis

D. Thrombocytopenia <100,000

15. The topical treatment in bullous pemphigoid you start with:

A. Clobetasol

B. Tacrolimus
C. Mupirocin

D. Cyclosporine

16. Scl-70 ANA is characteristic for:

A. SLE

B. Systemic sclerosis

C. Mixed connective tissue disease

D. DLE

17. Correct answers about pemphigus are:

1. The antibodies in pemphigus vulgaris are IgG

2. We observe mucosal involvement in pemphigus foliaceus

3. We start the treatment with oral antibiotics

4. The first line therapy is systemic prednisone with azathioprine

A. 1,3

B. 3,4

C. 1,2,4

D. 1,4
18. Nikolsky’s sign is presented in:

A. Pemphigus vulgaris

B. Generalized pustular psoriasis

C. Lichen planus

D. A + B are correct

19. In SLE you can observe:

1. Leukopenia
2. Lymphocytosis
3. Anemia
4. Elevated complement components (C3,C4)

A. 1,3

B. 1,2,3

C. 2,3,4

D. 3,4

20. Correct answers about erysipelas are:

1. The disease is caused by Streptococcus Pyogenes

2. Erysipelas can be recurrent

3. The typical location is lower leg (shins)

4. We treat it with oral steroids


5. Erysipelas never starts with fever

A. 1,2,3

B. 2,3,4,5

C. 2,3,4

D. 1,4,5

Random Credit 2
1. Streptococcus pyogenes is NOT responsible for:
A. Erysipelas
B. Impetigo contagiosa
C. Furunculus
D. Ecthyma
D. Fasciitis necroticans

2. What is NOT typical for erysipelas?


A. Lymphangitis
B. Fever
C. Erythema and swelling
D. Arterial ulcer
E. Localization on lower extremity

3. What is NOT a complication of erysipelas?


A. Thrombophlebitis
B. Elephantiasis
C. Skin necrosis
D. Phlegmon
E. Osteomyelitis
4. Borrelia Burgdorferi is responsible for:
A. Erythema multiforme
B. Allergic vasculitis
C. Erythema migrans (Erythema chronicum migrans)
D. Erythema nodosum
E. Ecthyma

5. For herpes zoster infection (shingles) is not true:

A. virus has latent phase


B. pain is prodromal symptom
C. skin lesions are unilateral
D. is common in children
E. mucous membranes may be involved

6. In lymphadenosis cutis benigna:


A. Face is common location
B. Surgical treatment is necessary
C. Skin changes are sclerotic
D. Fever is common
E. Antibiotic therapy is ineffective

7. Which complication is not observed in Herpes Zoster (Shingles) infection:


A. Nerve paralysis
B. Ocular nerve atrophy
C. Keratitis
D. Lymphangitis
E. neuralgia

8. Herpes simplex virus is not responsible for:


A. Eczema herpeticum kaposi
B. Ulcerative pharyngitis
C. Meningitis in infants
D. Herpes labialis
E. Herpangina

9. Molluscum contagiosum is not:


A. Viral infection
B. Pruritic disease
C. Topically treated (5% Potassium Hydroxide)
D. Common in atopic children
E. Self-limited disease

10. Which is false:


A. Human papilloma virus - warts
B. Herpes simplex virus - eczema herpeticum
C. Herpes simplex virus - erythema multiforme
D. Varicella zoster virus - shingles
E. Molluscum contagiosum virus - hand, foot and mouth disease

11. Pityriasis versicolor


A. Can be misdiagnosed as vitiligo
B. Can be misdiagnosed as scalp psoriasis
C. Affects intertriginous areas
D. All answers are wrong

12. Mark the wrong answer about different types of tinea:

A. Zoophilic dermatophytes cause more severe inflammation than anthropophilic


B. itchy interdigital maceration particularly in the third/ fourth interspace may occur on feet
C. diffuse dry scaling of the soles may occur on the feet
D. pustules and vesicles may be seen in tinea corporis
E. in contrast to candida it never affects groin

13. Onychomycosis

A. can be treated with systemic drugs without fungal tests if clinical signs are obvious in severe
cases (All Nails involvement)
B. topical antifungal are effective in most of the cases
C. tinea pedis is present on diagnosis in most of the cases
D. in cases caused by dermatophytes infection starts with proximal portion of the nail
E. pustules under the nail plate are usually present

14. Scabies

A. itching is more pronounced during mornings (nights)


B. oral Permethrin is effective (topical)
C. proper hygiene (regular showering) prevents the disease
D. typical lesions can be found on palms and Soles in children
E. pustules are typically present

Random Credit 3
1. Typical for leukoclastic vasculitis is
A. Small vessel destruction
B. Palpable purpura
C. Often appears after infection
D. Lower leg involvement
E. All above

2. Pyoderma gangrenosum can be associated with


A. Chronic ulcerative colitis
B. Diabetes mellitus
C. Hypertension
D. Psoriasis
E. Urticaria

3. Alopecia areata
A. Is an irreversible, scarring form of hair loss
B. Has a variable and unpredictable clinical course
C. Never progresses to complete loss of scalp hair
D. Does not involve body hair
E. Systemic antifungal can be effective

4. Scarring (cicatricial) alopecia can be present in


A. Lichen planus, cutaneous lupus erythematosus, alopecia areata
B. Lichen planus, alopecia areata, androgenic alopecia
C. Androgenic alopecia, alopecia areata, lichen planus
D. Alopecia areata, cutaneous lupus erythematosus, frontal fibrosing alopecia
E. Frontal fibrosing alopecia, lichen planopilaris, cutaneous lupus erythematosus

5. Koebner phenomenon occurs in


E. Pemphigus foliaceus
F. Lichen planus
G. Erythrasma
H. Acne
I. Rosacea

6. Patergy can occur in


E. Alopecia areata
F. Vitiligo
G. Pyoderma gangrenosum
H. Allergic vasculitis
I. Rosacea
7. To confirm the diagnosis of allergic vasculitis (hyperergic purpura) one can perform
A. USG doppler of leg arteries and veins
B. Search for circulating IgA antibodies (indirect immunofluorescence)
C. Histopathology and direct immunofluorescence
D. Prick test
E. All of the above

8. Acne vulgaris can be treated with


A. Topical isotretinoin
B. Systemic isotretinoin
C. Clindamycin with benzoyl peroxide
D. B and c are correct
E. All are correct

Random Credit 4

1. Psoriasis rarely effects


A. elbows
B. knees
C. scalp
D. face
E. sacral area

2. Re-Puva means:
A. puva therapy of resistant lesions
B. remission of psoriasis after puva therapy
C. relapse after puva
D. therapy with retinoids and puva
E. repeated sessions of puva
3. For chronic plaque psoriasis
A. Pustules and plaques are typical lesions
B. lesions are poorly demarcated (well demarcated)
C. nikolsky sign is positive (in generalized pustular psoriasis)
D. auspitz sign is negative (positive)

4. Which systemic therapy should not be used in skin psoriasis


A. Methotrexate
B. cyclosporine
C. steroids
D. biological
E. all can be used

5. Target antigen for autoantibodies in pemphigus foliaceus is


A. desmoglein 1
B. desmoglein 2
C. desmoglein 3
D. desmocollin 1
E. desmocollin 3

6. It is not true that pemphigus vulgaris


A. effects mucous membranes and skin
B. usually starts in Adulthood
C. is relatively frequent in Mediterranean area
D. requires the therapy with high doses of prednisone and azathioprine
E. is usually curable in one year

7. Bullous pemphigoid:
A. mainly affects mucous membranes (rarely affect mucous membranes)
B. usually starts in childhood (elderly)
C. is paraneoplastic disease
D. requires high dose of prednisone
E. IGG and/or C3 can be found in basal membrane Zone

8. Which answer describes being morphea (localized scleroderma) is not true:


A. more frequent in adults
B. Raynaud's phenomenon is positive
C. likely associated with borrelia burgdorferi
D. unrelated to systemic Scleroderma
E. negative anti-dsdna antibodies

9. Which variant of morphea is related to flexion contractures:


A. linear morphea
B. plaques like morphea
C. circumscribed morphea
D. macular morphea
E. none of the above

10. Lilac ring is characteristic for:


A. lichen planus
B. Vitiligo
C. superficial fungal infections
D. candidiasis
E. morphea

11. Which symptom is not present in Crest syndrome


A. calcinosis
B. pulmonary fibrosis
C. sclerodactyly
D. telangiectasia
E. esophagus dysfunction

12. Which skin lesion is not observed in a course of systemic lupus erythematosus
A. butterfly rash
B. scars
C. wheals
D. pustules
E. blisters

13. Discoid cutaneous lupus erythematosus (DLE) which answer is not true:
A. May transfer to SLE
B. children may be affected
C. skin lesions May evolve to ulcers
D. facial involvement is typical
E. exacerbation after sun exposure

14. Following characteristic features for Subacute cutaneous lupus erythematosus (SCLE) are
true except for:
A. more frequent than women
B. psoriasiform skin lesions are possible
C. scarring is typical
D. anti-Ro/SS-A and anti-La/SS-B are common
E. annular skin lesions are possible

15. False statement for dermatomyositis is:


A. may be associated with malignancy (adult form and not juvenile form)
B. immunological markers are RNP antibodies
C. level of creatinine phosphokinase is elevated
D. photosensitivity may occur
E. muscle weakness affecting proximal limb girdle
16. Which of skin lesions are not observed in a course of limited systemic scleroderma:
A. Radial furrowing around the lips
B. microstomia
C. mask-like face
D. intensive Raynaud phenomenon
E. Central skin involvement

17. Systemic Scleroderma which of the following answers is not true


A. ANA are present in majority of the patients
B. Ro/SSA antibodies are characteristic
C. esophagus is commonly affected
D. Renal Crisis is rare
E. pulmonary fibrosis may occur

18. Which feature is characteristic for epidermolysis bullosa acquisita:


A. flaccid blisters (pemphigus)
B. predominance in past middle age
C. wooly hairs
D. palmoplantar hyperkeratosis
E. involvement of traumatized areas

19. What is not characteristic for pemphigus foliaceus:


A. nikolsky's phenomenon
B. oral erosions
C. flaccid blisters
D. predominance in middle age
E. hypersensitivity to UV

20. Mucous membrane (cicatricial) pemphigoid may be complicated by:


A. neuralgia
B. blindness
C. sindactilia
D. deafness
E. Paralysis

21. Paraneoplastic pemphigus is characterized by:


A. severe mucosal involvement
B. Association with solid cancers
C. Association with non-melanoma skin cancers
D. presence of anti-desmoglein 1 antibody
E. rapid response to dapsone

22. Dermatitis herpetiformis is characterized by the presence of:


A. large blisters in traumatized areas
B. flaccid blisters in lower abdomen
C. vesiculobullous eruptions on extensor aspects of forearm
D. scars
E. non itching disseminated papules

23. Linear IGA bullous dermatosis in adults:


A. May clinically mimic bullous pemphigoid
B. requires high dose of prednisone
C. is characterized by the presence of granular IGA deposits in dermal papillae
D. is associated with gluten enteropathy (dermatitis herpetiformis)
E. requires gluten-free diet (dermatitis herpetiformis)

24. Subacute cutaneous lupus erythematosus May mimic:


A. psoriasis
B. lichen planus
C. erysipelas
D. acne
E. erythema nodosum

25. A mother of a child with heart block should be screened for the presence of:
A. Lupus anticoagulant
B. circulating anti-DSG 1 antibodies
C. anti-Dsg 3 antibodies indirect immunofluorescence
D. dsDNA antibodies
E. anti-Ro and La antibodies

26. Not typical for dermatomyositis is:


A. heliotrope
B. V-sign
C. cicatricial alopecia
D. gottron's sign
E. elevated liver enzymes (AST and ALT)

Random Credit 5

1. ABCD rule in clinical assessment of nevus has low sensitivity for diagnosis of
A. Superficial spreading melanoma
B. Nodular melanoma
C. Lentigo maligna melanoma
D. Acral lentiginous melanoma
E. Is equally very sensitive (>90%) for all common variants of melanoma
2. Melanoma can evolve from
A. Normally looking skin
B. Common melanocytic nevus
C. Atypical (dysplastic) melanocytic nevus
D. A and C are true
E. All answers are correct

3. Breslow scale is used to describe


A. Skin phototype
B. The result of a path test
C. Histopathologically assessed melanoma thickness
D. Describe a wheal during prick testing
E. The level of skin sclerosis in systemic sclerosis

4. The most common variant of melanoma is


A. Lentigo maligna melanoma
B. Superficial spreading melanoma
C. Nodular melanoma
D. Acral lentiginous melanoma
E. Amelanotic melanoma

5. Mycosis fungoides
A. Has a worse prognosis than Sezary Syndrome (the other way around)
B. Chronic sun exposure is a risk factor for MF
C. Cytostatic drugs are the first line therapy for the erythematosus stage
D. Retinoids can be helpful (bexarotene)
E. Is never erythrodermic

6. Which of the following technique should be used to histopathologically confirm or exclude the
clinical diagnosis of melanoma on the extremities?
A. Shave biopsy
B. Punch biopsy
C. Excisional biopsy (fusiform, along the skin tensionlines with 1-2 mm margin)
D. Excisional biopsy (fusiform, along the axis of extremities with 1-2 mm margin)
E. Excisional biopsy (fusiform, along the axis of extremities with minimum 1 cm margin)

7. Squamous cell carcinoma


A. Is more common that basal cell carcinoma
B. It is usually located on shoulder (common on the lower lip and ear)
C. The course is better when it affects mucosa
D. Surgery should be left only for advanced stages (with metastases)
E. Kidney transplant patients have a high risk of SCC development

8. Bowen disease
A. Can be treated with cryotherapy
B. Is a variant of lentigo
C. Is located on mucosal membranes
D. Is primarily malignant
E. Telangiectasias are present typically in large lesions

9. Actinic keratosis
A. Can be treated with oral 5-FU (Topical)
B. Intralesional triamcinolone
C. Photodynamic therapy
D. Topical imiquimod
E. C and D are correct

Random Credit 6

1. Characteristic feature for Erythema multiforme syndrome is


A. Healing with scars
B. Dorsa of hands, palms and soles involvement
C. Skin lesions are asymmetric (symmetric)
D. More frequent in children
E. ANA positive

2. Toxic epidermal necrolysis, which of the following answers is TRUE


A. Nikolsky sign is negative
B. More common in children
C. 50% of patients report no drug use
D. More than 30% epidermal detachment
E. Nails are not involved

3. Raynaud phenomenon is observed in the following diseases EXCEPT


A. Systemic scleroderma, dermatomyositis
B. SLE, Sjøgren Syndrome
C. Dermatomyositis, Rheumatoid Arthritis
D. SLE, Morphea generalisata
E. Sjøgren syndrome, CREST

4. Gold standard in treatment of pustular psoriasis is


A. Methotrexate
B. Dithranol
C. Acitretin
D. Cyclosporine
E. TNF alpha antagonists

5. Psoriasis vulgaris type I, which of the following is not true


A. Eruptions usually after infections
B. Often family occurrences
C. Onset in young adults
D. Often chronic stable type
E. Association with HLA-Cw6
6. Psoriasis epidermis is characterized by
A. Decreased number of keratinocytes
B. Elongation of cell cycle
C. Parakeratosis
D. Decreased mitotic rate
E. Dyskeratosis

7. Which of the following factors can not trigger the eruption of psoriasis
A. Physical trauma
B. Alcohol
C. Infections
D. Medications
E. None of the above

8. How long is contraception recommended after discontinuation of acitretin


A. 1 month
B. 2 months
C. 6 months
D. 1 year
E. 2 years

9. 40 year old woman with erythematosus nodes on the shines and hilar lymphadenopathy. What do
you suspect?
A. Lofgren syndrome
B. Graham-little syndrome
C. Leiner disease
D. Erysipelas
E. Diabetes

10. Which of the following can be drug induced?


A. Erythema multiforme
B. Lyell syndrome
C. Pemphigus vulgaris
D. Erythema fixum
E. All answers correct

11-12 missing

13. Steven Johnson syndrome is a severe form of erythema multiforme characterized by


A. Oral mucous membrane involvement
B. Potential widespread skin involvement
C. Blistering
D. Ocular and genital involvement
E. All correct

14. False about erysipelas is


A. The affected area is hot and tender
B. It involves both the cutaneous and subcutaneous tissue
C. Most common localization are the face and lower leg
D. If recurrent it may lead to chronic lymphedema
E. The most common etiology is Staph aureus

15. True about impetigo


A. It is more common in adults than children
B. The skin lesions are small erosions with crust
C. The skin lesions are small ulcers with thick crust
D. The most common etiology is S. pyogenes
E. Vesicles and bullae are never its clinical manifestations

16. False about irritant contact dermatitis is


A. May be treated with topical glucocorticoids
B. Clinical manifestations are erythema, vesicles and erosions
C. Patch tests are often positive
D. Symptoms may occur within seconds after exposure to some substances
E. Often erythema is the first manifestation

17 missing

18. Which of the following factors can not trigger the eruption of psoriasis
A. Physical trauma
B. Alcohol
C. Infections
D. Medications
E. None above

19. Discoid cutaneous lupus erythematosus (DLE), which of the following is NOT true?
A. May develop SLE in 1-5% of cases
B. Children may be affected
C. Skin lesions may evolve in ulcers
D. Facial involvement is typical
E. Exacerbation after sun exposure

20. Following characteristic features for subacute cutaneous lupus erythematosus (SCLE) are following
EXCEPT
A. More frequent in women
B. Psoriasisform skin lesions are possible
C. Scarring is typical
D. anti-Ro/SS-A and anti-La/SS-B are common
E. Annular skin lesions are possible

21. Which of skin lesions are not observed in SLE


A. Butterfly rash
B. Discoid lupus erythematosus
C. Urticarial vasculitis
D. Pustules
E. Bullous skin lesions

Random Credit 7
1. Sign the wrong answer about molluscum contagiosum
a) The primary lesion are skin-colored papules
b) Mollusca usually undergo spontaneous regression
c) Differential diagnosis includes condylomata
d) Topical antibiotic can be effective in localized
e) It can affect both children and adults

2. Borreliosis should be treated with:


a) Antibiotics for 7 days
b) Antiviral drugs for 7 days
c) Antibiotics for 21 days
d) Antiviral drugs for 21 days
e) Non Steroidal anti-inflammatory drugs

3. The following are characteristic for zoster except:


a) Prodromal itching
b) Conjunctivitis
c) Delayed hemiparesis
d) Folliculitis
e) Dermatomal involvement

4. Which phenomenon is characteristic for flat warts:


a) Auspitz' sign
b) Nikolsky's sign
c) Darier's sign
d) Koebner phenomenon
e) "oil spots"

5. Psoriasis rarely affects:


a) Elbows
b) Knees
c) Scalp
d) Face
e) Sacral area

6. Psoriasis:
a) PUVA therapy of resistant ?
b) Remission of psoriasis after PUVA
c) Relapse after PUVA
d) Therapy with retinoids and PUVA
e) Repeated sessions of PUVA

7. Which systemic skin therapies shouldn't be used in psoriasis:


a) Methotrexate
b) Cyclosporine
c) Steroids
d) Biological
e) All can be used

Random Credit 8
1- In which skin disease we don’t use photochemotherapy?

a) Atopic dermatitis
b) Granuloma annulare

c) SLE

d) Morphea

2- Raynaud syndrome may be present in

a) Systemic scleroderma

b) SLE

c) Rheumatoid dermatitis

d) All of the above

3- Nikolsky’s sign is characteristic for

a) Pemphigus vulgaris

b) Toxic epidermal necrolysis

c) All of the above

d) None of the above

4- Risk factors for erysipelas are not

a) Drug and alcohol abuse

b) Positive family history

c) Chronic lymphedema

d) Cirrhosis, diabetes
5- For sezary syndrome is not characteristic

a) Hyperkeratosis of palms and soles

b) Erythroderma

c) Peripheral lymphadenopathy

d) Atypical lymphocyte B in the blood

6- Find the correct answer

a) ABCD rule is used in atopic dermatitis

b) Tinea pedis is differentiated with impetigo

c) Tinea barbae only affects women

d) There is no nail involvement in lichen planus

7- Find wrong answer

a) Genital candidiasis usually represent overgrowth of colonizing infection

b) Topical treatment in tinea unguium is usually sufficient

c) The use of emollients is a basic treatment in every case of atopic dermatitis

d) Dermatitis perioralis can be caused by topical glucocorticosteroids

8- We don’t use in topical treatment of seborrheic dermatitis

a) Selenium sulfide

b) Glucocorticosteroids

c) Gentamicin
d) Ketoconazole

9- Which pair doesn’t match

a) Psoriasis vulgaris – papules

b) Zoster – vesicle

c) Erythema multiforme – macule

d) Eczema – nodule

10- We don’t use the systemic treatment of psoriasis

a) Retinoids

b) Glucocorticosteroids

c) UVA radiations

d) Cyclosporine A

11. White dermograph is characteristic for?

a) Psoriasis

b) Atopic dermatitis

c) Pemphigus

12- What does not affect oral mucosa

a) Pemphigus vulgaris

b) Pemphigus foliaceus
c) Pemphigoid

d) Lichen planus

13- face acne – do not use steroid

14- isotretinoin side effects – teratogenic, blindness, increase lipid profile, photosensitivity,
psychiatric SE

15- not differential diagnosis of rosacea – dermatitis herpertiformis

16- how do you differentiate contact dermatitis and irritant dermatitis – irritant is confined at site
of exposure

17- which is related to photosensitivity – porphyria

18- area of childhood atopic dermatitis – antecubital, popliteal fossa, face

19- acantholysis is related with – pemphigus vulgaris

20- not characteristic for herpes zoster – not symmetry

Random Credit 9
1. Indicated in psoriasis are: which is NOT true
Ans: oral steroids
2. Primary lesion in psoriasis is:
Ans: papule
3. Ro/La antibodies are immunologic marker for:
Ans: SCLE
4. In which disease skin lesions resolve with scaring:
Ans: DLE
5. In atopic dermatitis we can do several diagnostic tests-which is NOT true:
Ans: Immunopathology
6. Desmoglein 3 is an antigen in:
Ans: Pemphigus Vulgaris
7. Morphea:
Ans: Skin lesion are surrounded by lilac ring
8. PUVA is NOT indicated in:
Ans: SLE
9. Psoriatic arthritis-match false answer:
Ans: Always coexist with skin lesion

10. Lichen planus-match false answer:


A. mucosal lesions can be located in oral cavity
B. viral hepatitis can be a concomitant disease
C. typical treatment are topical retinoids
D. Wickham striae can be present on top of lesions

11. Typical for pemphigoid in direct immunofluorescence is:


A. IgG at the basement membrane zone
B. intracellular IgG
C. only C3 component factor at the basement membrane zone
D. Immunofluorescence is not used in the diagnosis om pemphigoid

12. Pemphigoid treatment can be based on-match FALSE answer:


A. methotrexate
B. tetracycline
C. prednisone
D. retinoids

15. Acne fulminans is a:


A. flare of acne, usually with severe folliculitis, nodules and cysts in tropical climates
B. persistent acne in hirsute female
C. acute acne with cysts, nodules, erosions with fever, general arthralgia in teenage boys
D. mild acne usually in young…...

13. Dermatomyositis-which is FALSE:

Ans: in juvenile type there is no Gottron sign

14. Alopecia can be found in :


Ans: SLE, DLE, Lues
15. Common cause of urticaria:
Ans: food+drug
*If it’s asking contact dermatitis: then the common cause is hapten
Hapten: not full allergen, must be bound to protein first
16. Impetignisation means:
Ans: secondary bacterial infection of skin lesion
17. Erysipelas can be complicated by: which is FALSE:
Ans: Folliculitis
18. Atopic dermatitis found in:
Ans: flexor side of knees and elbows
19. Autoantigen of pemphigoid:
Ans: BP-180
20. Treatment for Rosacea:
Ans: oral and topical metronidazole
21. PUVA treatment is indicated for:
Ans: Psoriasis, mycoses fungoides, morphea
22. PUVA side effects:
Ans: skin cancer, cataract, skin erythema
23. Differential diagnosis for Tinea pedum: which is FALSE:
Ans: Atopic dermatitis
24. Acute urticaria treatment:
Ans: oral antihistamines, no topical treatment
25. Erythrodermia-typical cause:
Ans: Psoriasis, mycosis fungoides, atopic dermatitis
*Erythroderma is the term used to describe intense and usually widespread reddening of the
skin due to inflammatory skin disease. In Rosacea not whole face is red so this is one of the
causes!
26. Pemphigus vulgaris is characterized by:
Ans: Nikolsky sign
26. Dermatomyositis marker:
Ans: Jo-1
27. Major criteria for atopic dermatitis:
Ans: typical locations, prolonged or recurrent course of disease, intensive itch, coexistence with
other atopic diseases
28. Kobner phenomenon can be found in:
Ans: Psoriasis/lichen planus/common wart/plane wart
29. Same as Q10
30. Pemphigus foliaceus:
Ans: presence of IgG antibodies anti desmoglein 1

Random Credit 10
21. Rosacea
A. is a bacterial disease of adulthood
B. in the last stage rhinophyma is observed
C. steroids are the best treatment
D. there is no eye involvement

24. Pityriasis rosea is associated with reactivation of:


A. HIV
B. HHV-7
C. HHV-3
D. HSV

27. In management of SLE which drug is not in use


A. hydroxychloroquine
B. prednisone
C. tetracycline
D. leflunomide

31. The biopsy for DIF in vesiculobullous diseases should be taken from
A. center of blister
B. perilesional skin
C. erosions
D. site of biopsy is not important

32. Which antibodies are found in DIF in dermatitis herpetiformis


A. linear IgA
B. Linear IgG and C3
C. IgG in the epidermis
D. IgA in dermal papillae

33. The premalignant lesion of skin is not


A. actinic keratosis
B. xeroderma pigmentosum
C. granuloma anulare
D. Bowen disease

34. Indicate true statement


A. UV is the evidenced oncogenic factor in development of skin cancer
B. non-surgical treatment of BCC do not have to be preceded wi….
C. SCC is the most frequent non-melanoma skin cancer
D. patients with actinic keratosis do not need UV protection

35. Primary lesion in acne vulgaris


A. papule
B. pustule
C. comedone
D. nodule

37. Erythroderma can occur in following diseases - indicate false


A. Sezary syndrome
B. psoriasis
C. atopic dermatitis
D. scleroderma

Random Credit 11

From Q10 onwards, use this key. Otherwise, normal MCQ


A. All true or false
B. 1,2,3 true
C. 1,2,4 true
D. 1,3,4 true
E. 2, 3, 4 true

1. What treatment is effective in keratitis parenchymatosa


1. Procaine penicillin
2. Crystalline penicillin
3. Tetracyclines
4. Macrolides
5. Steroid

2. Which positive test will prove syphilis in a newborn


1. VDRL
2. TPHA
3. FTA
4. Captia IgM
5. RPR

3. The risk of infecting fetus by HIV women given retroviral therapy is:
1. 100%
2. 50%
3. 30%
4. 2%
5. None

4. Meningovascular syphilis and cardiovascular syphilis do not have in common:


1. Endarteritis obliterans
2. Belong to later syphilis
3. Belong to secondary syphilis
4. Good response to penicillin teatment
5. Which test is quantitative and not cardiolipin
1. RPR
2. USR
3. VDRL
4. FTA
5. TPHA

6. Serology is not useful in:


1. Latent early syphilis
2. Latent late syphilis
3. Concomitant syphilis and HIV
4. Chlamydia trachomatis inf.
5. Congenital syphilis

7. The shortest incubation time is in:


1. Syphilis
2. Chlamydiosis
3. Gonorrhea
4. HIV
5. HPV

8. The highest risk of infection after one sexual contact:


1. Syphilis
2. HIV
3. Chlamydiosis
4. HPV
5. HSV

9. Acquired syphilis and congenital syphilis do not have in common:


1. Papular rash
2. Condylomata lata
3. Bullous rash ( the only bullous rash seen in syphilis)
4. ….

…..Missing Qs

47. Congenital and acquired syphilis do not have in common:


1. Condylomata lata
2. General lymphadenopathy ?
3. Alopecia
4. No primary score

48. Congenital HIV:


1. Risk is 20% when mother is treated (2%)
2. Detected by group of lymphocytes
3. Detected by serology
4. Uncommon

…...Missing Qs

56. What is eczema herpeticum (en eller fler er rett)


1. Disseminated herpes, occuring in atopic dermatitis, pemphigus vulgaris, thermal burns
2. Herpes characterized by large grouped vesicles, often on the trunk
3. Punched out lesions, with erosion and crust on face, neck
4. occur in pregnancy

57. Most common herpes infection in childen?


….Herpectic gingivo stomatitis

58. The clinical stages in herpes zoster:


Prodrome: neuritic pain, paresthesia
Acute: vesiculation; pruritic; lasts 3-5 days
Chronic: after healed lesion, postherpetic neuralgia
59. Which dermatomal part of body are most common affected in Zoster?
Thoracic

60. Koebner phenomenon occur in the following except:


1. Psoriasis
2. Verrucae vulgaris
3. Seborrheic dermatitis
4. Lichen planus
5. Verruca plana

61. Warts commonly found on hands and the face of children: Plante Warts

62. What are the main criteria for atopic dermatitis?


1. Chronic recurrent
2. Family history; conjunctivitis, asthma, dermatitis, rhinitis
3. Typical locations; FLEXURAL areas,
4. Itch, dry

63. What is the primary lesion in atopic dermatitis?


Papule

64. What are secondary lesions in atopic dermatitis? (T/F)


1. Ulcerations F
2. Lichenification T
3. Alopecia (eyebrows) T
5. Secondary candida

69. Which type of psoriasis involves mucous membranes?

A. Guttate psoriasis
B. PPP
C. Generalised pustular (von zumbusch)
D. CHronic plaque psoriasis

70. What is not a good treatment for psoriasis:


A. Salicylic acid

B. Topical dithranol

C. PUVA - oral isotretinoin


D. systemic ccs

71. What can we not find on a laboratory exams in mycosis fungoides


A. Pautrier’s microabscess
B. Eosinophilia and increased WBC
C. Buffy coat: Sezary cells
D. HLA - DR8

72. What is not present in Sezary syndrome


A. Universal erythroderma
B. Poikiloderma
C. Peripheral lymphadenopathy
D. Infiltrates of atypical lymphocytes

74. Which one is false


A. Alopecia is associated with Hashimoto’s thyroiditis, vitiligo, myasthenia gravis
B. Alopecia is an autoimmune disease
C. Psoriasis commonly causes alopecia

D. Seborrheic dermatitis on the scalp can be called dandruff

75. In which disease can we find Gottron’s papules


Dermatomyositis and SLE
76. Lyell syndrome which is not true?
A. Minor form of erythema multiforme
B. Nikolsky sign
C. fever higher than in SJS
D. eyelashes and nails may be shed

77. Erythema nodosum (T/F)


A is a panniculitis T
B. Most commonly on pretibial area T
C. Heal with scarring F
D. May be accompanied by fever and arthralgia T

2018 FINAL

1. Which antinuclear antibody is characteristic for renal involvement systemic lupus erythematosus?
A. Anti-histone
B. Anti - U1 - RNP
C. Anti-dsDNA
D. Anti-Sm
E. C and D

2. All of the below are clinical criteria of systemic lupus erythematosus with the exception of:
A. High complement
B. Thrombocytopenia
C. Hemolytic Anemia
D. Oral ulcers
E. Discoid lupus erythematosus lesions

3. AGEP (Acute Generalized Exanthematous Pustulosis) is a:


A. type of drug eruption
B. bacterial infection
C. viral infection
D. genodermatosis
E. connective tissue disease

4. What is the first line oral treatment for discoid lupus erythematosus?
A. Chloroquine
B. Belimumab
C. Acitretin
D. Cyclophosphamide
E. Etanercept

5. Streptococcus pyogenes is NOT responsible for:


A. Erysipelas
B. Impetigo contagiosa
C. Furunculus
D. Ecthyma
D. Fasciitis necroticans

6. What is NOT typical for erysipelas?


A. Lymphangitis
B. Fever
C. Erythema and swelling
D. Arterial ulcer
E. Localization on lower extremity

7. What is NOT a complication of erysipelas?


A. Thrombophlebitis
B. Elephantiasis
C. Skin necrosis
D. Phlegmon
E. Osteomyelitis
8. Borrelia Burgdorferi is responsible for:
A. Erythema multiforme
B. Allergic vasculitis
C. Erythema migrans (Erythema chronicum migrans)
D. Erythema nodosum
E. Ecthyma

9. HSV infection may provoke:


A. Erythema multiforme
B. Periorbicular dermatitis
C. Erythema chronicum migrans
D. Perioral dermatitis
E. AGEP (Acute Generalized Exanthematous Pustulosis)

10. In lymphadenosis cutis benigna (borrelial lymphocytoma):


A. Scalp is common location
B. Surgical treatment is not necessary
C. Skin changes are sclerotic
D. Fever is common
E. Antibiotic therapy is ineffective

11. In the early stage of Borrelia burgdorferi infection:


A. Skin lesions are never disseminated
B. Nephritis is common
C. Flu-like symptoms may occur
D. Erythematosus lesion is painful
E. ACA are present

12. Indicate the lesion that is NOT characteristic for acne:


A. Nodule
B. Cyst
C. Papule
D. Blister
E. Pustule

13. Indicate a proper treatment for inflammatory acne (papulo-pustular acne):


A. Combined topical retinoid and benzoyl peroxide
B. Combined topical retinoid and topical antibiotic
C. Combined topical antibiotic and benzoyl peroxide
D. Combined topical retinoid and systemic tetracycline
E. All answers are correct

14. Which of the following drugs should NOT be used for the treatment of papulopustular rosacea:
A. Topical corticosteroid
B. Topical ivermectin
C. Topical azelaic acid
D. Topical metronidazole
E. Systemic tetracycline

15. Which of the following should NOT be used as a topical acne treatment:
A. retinoids
B. benzoyl peroxide
C. neomycin
D. clindamycin
E. salicylic acid

16. Indicate the treatment that is NOT used as topical treatment for psoriasis:
A. Topical crisaborole
B. Topical calcipotriol + betamethasone
C. Topical anthralin (cignoline)
D. Topical very potent steroid
E. Topical tar

17. Mark the drugs that are NOT used as systemic treatment for chronic plaque psoriasis:
A. Ustekinumab (anti-IL 12/23), Acitretin, Cyclosporin A
B. Rituximab (anti-IL-20), Cyclophosphamide
C. Methotrexate s.c, Apremilast (PDE4 inhibitor)
D. Infliximab (anti-TNFalpha), Secucinumab (anti IL-17)
E. All of the above can be used

18. Nikolsky sign is characteristic for:


A. Guttate psoriasis
B. Chronic plaque psoriasis
C. Inverted psoriasis
D. Palmo-plantar pustulosis
E. Generalized pustular psoriasis ( von Zumbusch type)

19. Late onset psoriasis (Type II psoriasis)


A. Familiar occurrence is easy recognizable
B. Guttate type lesions are characteristic for this variant
C. Typical exacerbations usually occur after fungal infections
D. HLACw6 is typical
E. All answers are wrong

20. Systemic retinoids are first-line treatment for:


A. Scalp psoriasis
B. Nail psoriasis
C. Palmo-plantar pustulosis
D. Psoriatic arthritis
E. All answers are wrong

21. Dark typical localizations of lesions in chronic plaque psoriasis:


A. Scalp, sacral
B. Acral, sun-exposed
C. Flexural, seborrheic
D. Oral mucosa, dorsal hands
E. All are correct

22. Mark common findings for DLE, SCLE, and systemic SLE:
A. Presence of skin lesions
B. Hypersensitivity to UV light
C. Immunoglobulin deposits at the dermo-epidermal junction (lupus band)
D. A, B, C are correct
E. All answers are wrong

23. Skin findings in SLE do not include:


A. mucosal erosions
B. DLE lesions
C. Perioral dermatitis
D. Erythema of fingers, foot soles and nails folds
E. Diffuse alopecia

24. Morphea
A. Methotrexate may be used for the treatment
B. Is a synonym of limited systemic sclerosis
C. Raynaud phenomenon is usually present
D. Atrophy of alae nasi is typical for this disease
E. ACA (anti-centromere antibodies) are usually present

25. Treatment of systemic sclerosis includes:


A. Sildenafil
B. Nifedipine
C. Alprostadil
D. Methotrexate
E. All above can be used

26. Lichen planus


A. Reticular lesions on buccal mucosa predispose to cancer (squamous cell carcinoma)
B. Oral erosions are always present
C. Pruritus is present in minority of patient with skin involvement
D. Vulvo-vaginal form is usually erosive
E. May be associated with Helicobacter infection

27. ACR/EULAR criteria for the classification of systemic sclerosis DO NOT include evaluation of:
A. Erythema
B. Skin hardening of the hands
C. Sclerodactyly
D. Erosive lesions/ulcerations on fingers
E. Telangiectasia

28. 30-year old patient with severe itching that gets worse at night, disseminated papules and similar
symptoms in his wife is like to suffer from:
A. Secondary syphilis
B. Lichen planus
C. Lichen plano-pilaris
D. Scabies
E. Dermatitis herpetiformis

29. What color of cutaneous lesion is characteristic for Microsporum infection in Wood’s lamp light:
A. pale yellow
B. blue green
C. coral red
D. light green
E. black

30. Athlete’s foot is a synonym for:


A. plantar warts
B. tinea pedis
C. dyshidrotic eczema of the foot
D. mechanical trauma of the foot
E. diabetic foot

31. What is NOT characteristic for Tinea capitis caused by zoophilic fungi?
A. hair loss
B. skin inflammation
C. skin swelling
D. pustules
E. fluconazole as a first line therapy

32. What is NOT characteristic for onychomycosis:


A. nail fragility
B. oil staining (oil drop spot)
C. subungual hyperkeratosis
D. nail separation
E. nail thickening

33. ‘Id-reaction’ is defined as:


A. allergy to peanuts
B. allergy to nickel
C. irritant eczema
D. allergy to fungi infection
E. drug intolerance

34. Which of the following is NOT a predisposing factor for skin candidiasis:
A. diabetes mellitus
B. systemic antibiotic therapy
C. systemic antiviral therapy
D. systemic corticosteroids
E. obesity

35. What is characteristic for bullous pemphigoid:


A. onset in childhood (elderly)
B. scarring conjunctivitis
C. flaccid blisters
D. erythematous lesions
E. butterfly rash

36. What is characteristic for pemphigus foliaceus?


A. Tense blisters
B. Nail involvement
C. Scarring conjunctivitis
D. Reaction of antibodies with desmoglein 1
E. Reaction of antibodies with desmoglein 1 and 3

37. To make a diagnosis of dermatitis herpetiformis it is crucial to identify:


A. linear IgA in dermal papillae
B. granular IgA in dermal papillae
C. linear IgA along the basement membrane zone
D. linear IgA in intraepidermal spaces of the epidermis
E. granular IgA in intraepidermal spaces of the epidermis

38. Characterization of mucous membrane pemphigoid does NOT include


A. scarring conjunctivitis
B. scarring alopecia (Brunsting-Perry pemphigoid)
C. stricture of esophagus
D. nails thickening
E. Milia formation on the skin

39. Dorsal pterygium is characteristic for nail involvement in the course of:
A. psoriasis
B. onychomycosis
C. lichen planus
D. pemphigus vulgaris
E. candidiasis
40. Subacute cutaneous lupus erythematosus may mimic:
A. psoriasis
B. lichen planus
C. erysipelas
D. acne
E. erythema nodosum

41. In the case of heart-block in newborn child its mother should be screened for the presence:
A. lupus anticoagulant
B. circulating anti Dsg 1 antibodies
C. anti Dsg3 antibodies in direct immunofluorescence
D. dsDNA antibodies
E. the presence of Ro and La antibodies

42. The first line treatment of severe chronic spontaneous urticaria is:
A. cyclosporine A
B. plasmapheresis
C. second generation antihistamines
D. topical and systemic corticosteroids
E. omalizumab

43. The most common contact allergen in Europe and USA is:
A. gluten
B. nickel
C. chromium
D. fragrances
E. latex

44. Not typical for dermatomyositis is:


A. Heliotrope
B. V-sign
C. Cicatricial alopecia
D. Gottron sign
E. Elevated liver enzymes (AST and ALT)

45. Typical localization for seborrheic dermatitis are all EXCEPT:


A. Eyebrows
B. Nasolabial folds
C. Neck
D. Scalp
E. Sternal area

46. Collarette desquamation is characteristic for:


A. pityriasis rosea
B. dermatitis herpetiformis
C. pemphigus
D. atopic dermatitis
E. psoriasis

47. Keratoacanthoma
A. appears usually in older patients (above 40 years)
B. Spontaneously regress
C. Has crateriform shape with horny plug
D. I and II skin phototypes are risk factor for this disease
E. All above are correct

48. Indicate the treatment for pyoderma gangrenosum:


A. systemic corticosteroids
B. clindamycin and rifampicin
C. cephalosporins and fluoroquinolones
D. tetracyclines or macrolides
E. surgical excision of necrotic tissue and topical treatment
49. Which of the findings listed are NOT characteristic for allergic vasculitis:
A. vesicles filled with blood
B. palpable purpura
C. incerations
D. severe abdominal pain
E. painful nodules

50. Patient with dermatomyositis should be screened for:


A. pulmonary hypertension
B. fungal infection
C. neoplasm
D. underlying neurological disease
E. diabetes

51. Photodynamic therapy is for the treatment of:


A. actinic keratosis
B. tinea
C. melanoma
D. rosacea
E. chronic plaque psoriasis

52. What is NOT typical for erythema nodosum:


A. lesions usually appear on anterior part of shins
B. nodules are usually ulcerated
C. tenderness lesions on palpation
D. disease can be provoked by Yersiniosis
E. drugs can be causative agents

53. For the treatment of atrophic acne scars, one can use:
A. Cryotherapy
B. Fractional CO2 laser
C. Intralesional triamcinolone
D. Intralesional 5-Fluorouracil
E. IPL device (intense pulsed light)

54. In alopecia areata


A. Perifollicular pustules are present at the early stage of disease
B. Total hair loss never occurs
C. Nail changes are possible
D. Fever and malaise are present in acute phase
E. All answers are wrong

55. Mycosis fungoides


A. Usually requires systemic itraconazole
B. Hairs should be diagnosed with a Wood lamp
C. PUVA or UVB light can be used in the early stages of disease
D. Spontaneously regress within approximately 6 weeks
E. Skin biopsy is NOT necessary to confirm the diagnosis

56. What anatomical structures does the herpes simplex virus occupy while in the phase of infection?
A. Sacral sensory ganglia
B. Epidermis of the genital area
C. Genital area skin glands
D. Genital area subcutaneous layer
E. Cutis of the genital area

57. Congenital syphilis symptoms do NOT include:


A. Hutchinson’s teeth
B. Fournier’s teeth (mulberry molars)
C. Jarisch-Herxheimer-Lukasiewicz sign
D. Higoumenakis sign
E. Parrot’s scar

58. Buschke-Lowenstein’s large genital warts


A. HPV 6 or 11
B. HPV 13 or 32
C. HSV 2 or 1
D. HSV 6 or 11
E. HPV 33 or 66

59. Hoigne syndrome (a pseudo-anaphylactic reaction characterized by acute neurological and


psychiatric symptoms) is a side-effect of treatment with:
A. Intravenous crystalline penicillin
B. Intramuscular procaine penicillin
C. Oral phenoxymethylpenicillin
D. Intravenous benzathine benzylpenicillin
E. Intramuscular ceftriaxone

60. A positive KOH test outcome (vaginal discharge odour intensification after mixing with a potassium
hydroxide solution) is typical of:
A. Gonorrhea
B. Chlamydiasis
C. Trichomoniasis
D. Giardiasis
E. Bacterial vaginosis

61. Queyrat’s erythroplasia is:


A. Basal cell carcinoma in situ
B. A complication of untreated vaginal candidiasis
C. An anatomical variant of the glans penis
D. Histologically an equivalent of Bowen’s disease on the genital mucosa
E. A large variety of genital warts

62. Choose the INCORRECT answer:


Primary acute retroviral syndrome during HIV infection:
A. Begins typically 3-4 weeks after infection
B. Is difficult to diagnose as it occurs mostly still in the “serological window”
C. Occurs most commonly as the so-called mononucleosis-like-syndrome
D. Has unspecific symptoms, never causing skin lesions
E. Most commonly lasts 1-2 weeks

63. Early rash in secondary syphilis is:


A. typically localised symmetrically on the trunk and upper extremities
B. polymorphic
C. long-lasting (a few months)
D. a cause of scarring
E. typically localised on the hands and feet

64. Condylomata lata are lesions caused by infections with:


A. Treponema pallidum
B. HPV 6,11
C. HPV 16,18
D. HPV 5,8
E. HSV 1,2

65. The most frequent etiology of non-gonococcal urethritis (NOU) is:


A. Trichomonas vaginalis
B. Herpes virus
C. Beta hemolytic streptococcus B
D. Chlamydia trachomatis
E. Candida albicans

66. Which disease listed below is NOT typically transmitted by sexual contact?
A. gonorrhoea
B. syphilis
C. Chlamydia
D. condylomata acuminata of the penis (genital warts)
E. vaginal candidiasis
67. Urethritis, arthritis and conjunctivitis are part of:
A. Pinta syndrome
B. Hoigne syndrome
C. Stevens-Johnson syndrome
D. Reiter’s syndrome
E. Marino-Marini syndrome

68. A possible treatment of recurrent active genital herpes is:


A. Acyclovir in a dose of 5x200mg daily for 5 days
B. Ritonavir at a dose of 5x200mg daily for 5 days
C. Acyclovir in a dose of 5x200mg daily for 2 days
D. Acyclovir 800mg in a single dose
E. Ritonavir at a dose of 2x300mg per day for 3 days

69. Molluscum contagiosum:


A. Most commonly appears in children and young adults
B. Is caused by an HPV virus (MCV virus)
C. Typically occurs as a group of stalked or thread-like lesions, pink or greyish-white in color
D. Is treated with valacyclovir or acyclovir (Topical potassium hydroxide)
E. All answers are incorrect

70. Which of the below is NOT a common cause of vaginal discharge?


A. Syphilis
B. Gonorrhoea
C. HSV
D. HPV
E. Candida Albicans

71. Indicate a disease that CANNOT be a cutaneous drug eruption:


A. Erythema multiforme
B. Lichen planus
C. Atopic dermatitis
D. TEN
E. Urticaria

72. This what is commonly seen in patients with a history of atopic dermatitis:
A. Incontinentia pigmenti
B. Palmo-plantar pustulosis
C. Necrobiosis lipoidica
D. Isolated eyelid dermatitis
E. Dermatitis herpetiformis

73. Chronic urticaria is defined as urticaria with recurrent episodes lasting longer than:
A. 2 weeks
B. 6 weeks
C. 3 months
D. 6 months
E. 1 year

74. Associated signs of atopic dermatitis may include:


A. Keratoconus
B. Pityriasis alba
C. Xerosis
D. Anterior neck folds
E. All of the above

75. The epidermal barrier dysfunction in atopic dermatitis does NOT include:
A. Defect of filaggrin
B. Decrease in epidermal ceramides content
C. Alternations of the stratum corneum pH
D. Defect of desmoglein
E. Overexpression of the chymotryptic enzyme (chymase)
76. Second line treatment in chronic urticaria are all below except:
A. Leukotriene antagonists
B. Corticosteroids
C. Cyclosporin (3rd line)
D. Dapsone
E. UV light

77. The skin of a person with atopic dermatitis is more susceptible to:
A. Warts
B. Herpes simplex infections
C. Staphylococcus aureus infections
D. Candida spp. Infections
E. All of the above

78. The most common location of oral lichen planus is:


A. buccal mucosa
B. gingiva
C. tongue
D. hard palate
E. soft palate

79. Erythroderma:
A. is also known as erythrasma and is caused by Corynebacterium minutissimum
B. is a long-term skin condition characterized by facial redness and papules
C. is an acute streptococcal skin infection present typically with a skin rash
D. is characterized by generalized erythema (over 90%)
E. is one of the most common variants of psoriasis

80. Severe psoriasis can be treated with:


A. omalizumab
B. acitretin
C. tazarotene
D. oral corticosteroids
E. antimalarials

81. Mark the correct sentence about psoriasis:


A. Type 1 of the psoriasis have an earlier onset and more severe course
B. Type 1 of the disease is characterized by an involvement of oral mucosa
C. Type 1 of the disease is frequently preceded by urinary tract infection
D. In type II of the disease psoriatic lesions are present in offspring
E. Type 1 and II differ primarily in response to treatment

82. The treatment in patients with metastatic melanoma or advanced unresectable melanoma is:
A. Ipilimumab
B. Adalimumab
C. Vemurafenib
D. A, C correct
E. A, B correct

83. Sweet’s syndrome is characterized by:


A. something erythematous and edematous papules and plaques that are usually associated with pruritus
B. numerous small (<5mm), non-follicular, sterile pustules arise within large SOMETHING of edematous
erythema
C. fever, peripheral blood neutrophilia and painful erythematous plaques that favor the face and upper
extremities
D. erythematous macules in a symmetric distribution that sometimes become slightly palpable
E. none of the above

84. Diagnostic scoring systems for Drug Reaction with Eosinophilia and System Symptoms (DRESS) does
NOT include:
A. Peripheral neutrophilia
B. Lymphadenopathy
C. Circulating atypical lymphocytes
D. Peripheral hypereosinophilia
E. A, B correct

85. Diagnostic scoring system for toxic epidermal necrolysis does NOT include:
A. Age > 40
B. Cancer of hematologic malignancy
C. Serum glucose level
D. Serum IgE level
E. Serum bicarbonate level

86. The target chromophore in vascular lesions is:


A. oxyhemoglobin
B. water
C. melanin
D. A,B,C correct
E. All of the above

87. Cryosurgery is often used to destroy skin lesions such as:


A. actinic keratosis
B. melanoma
C. viral warts
D. A, C correct
E. All the above

88. CREST syndrome is characterized by:


A. calcinosis
B. erosive lesions of fingers
C. sclerodactyly
D. A,B,C correct
E. A,C correct

89. Criteria for the Classification for systemic sclerosis does NOT include:
A. pulmonary fibrosis
B. calcinosis
C. sclerodactyly
D. Erosive lesions of fingers
E. skin hardening on the hands

90. Order the layers of the epidermis from the bottom to the top:
A. stratum basale, stratum granulosum, stratum spinosum, stratum corneum
B. stratum basale, stratum spinosum, stratum granulosum, stratum corneum
C. stratum basale, stratum granulosum, stratum spinosum, stratum corneum
D. stratum corneum, stratum granulosum, stratum spinosum, stratum basale
E. stratum basale, stratum spinosum, stratum granulosum, stratum basale

91. Primary skin lesions are listed below, except for:


A. a macule
B. a papule
C. a crust
D. a vesicle
E. a wheal

92. A patient with skin phototype III:


A. rarely burns, tans easily
B. never burns, tans very easily
C. always burns, never tan
D. burns easily, tans with difficulty
E. has mild burns, tans gradually

93. Choose the WRONG answer. Erythema gyratum repens:


A. is more common in men
B. usually occurs after the seventh decade of life
C. is characterized by erythematous concentric lesions in a pattern resembling the grain of wood
D. is most commonly associated with lung cancer
E. has moderate associated with neoplasms
94. As a prognostic factor of melanoma we use a scale of:
A. PASI
B. BSA
C. Breslow
D. Fisher
E. Caspar

95. The best choice (from mentioned below) to treat superficial basal cell carcinoma is:
A. Clobetasol
B. Tacrolimus
C. Imiquimod
D. Isotretinoin
E. Pimecrolimus

96-100 BLURRY

2017 CREDITS

TEST 1

1. Nikolsky's sign 1 is characteristic for:

A. SSSS

B. Toxic Epidermal Necrolysis

C. Steven Johnson syndrome

D. Pemphigus vulgaris

E. All of the above


2. When a clinical and dermoscopic evaluation fails to clearly differentiate between atypical nevus and
melanoma, one should:

A. control the lesions every 12 months with dermoscope or better with wideodermoscope

B. perform a shave biopsy

C. perform incisional biopsy of the thickest part of the lesion

D. perform excisional biopsy with 1-3 mm margin

E. remove surgically lesion with 1 cm margin

3. Intense widespread redding of the skin, affecting over 90% of the body is called:

A. Erythema

B. Erythrasma

C. Erythroderma

D. Erysipelas

E. Erythrocytosis

4. What should be control during methotrexate therapy?

A. liver enzymes, morphology, lipids

B. blood pressure, creatinine, glomerular filtration rate

C. liver enzymes, glucose, creatinine

D. lipids, CA19-9, liver enzymes

E. There is no need for any control laboratory tests


5. Which type of allergic mechanism (by Galen and Coombs) is involved in the pathogenesis of allergic
urticaria?

A. Type 1

B. Type 2

C. Type 3

D. Type 4

E. Type 5

6. In atopic dermatitis we do not prescribe:

A. Allergen avoiding

B. Topical calcineurin inhibitors

C. Phototherapy

D. Chronic treatment with oral prednisone

E. Skin hydration 3-4 times a day with emollients

7. Primary skin lesions of lichen planus are not:

A. Pruritic

B. Elevated

C. Edematous

D. Shiny
E. Polyglonal

8. Patch tests with haptens help to establish the diagnosis of:

A. Atopic dermatitis

B. Urticaria allergic

C. Aspirin urticaria

D. Dermatitis herpetiformis

E. None of the above

9. In diagnosis of chronic urticaria one could perform:

A. Aspirin exposure test

B. Autologous serum skin test

C. Pressure test

D. All of the above

E. None of the above

10. Indicate specific autoantibodies that can occur in dermatomyositis:

A. ANA

B. Jo-1

C. Anti-Sm
D. ANCA

E. rRNP

11. Auspitz sign is typical for:

A. Lichen planus

B. Parapsoriasis

C. Psoriasis

D. Dermatomyositis

E. B and C

12. Which of the following diseases has the highest mortality rate?

A. Toxic epidermal necrolysis

B. DRESS syndrome

C. Acute generalized exanthematous pustulosis (AGEP)

D. Erythema nodosum

E. Steven Johnson syndrome

13. 85-year old male present with disseminated tense blisters on the trunk and limbs. DIF reveals linear
IgG and C3 deposits among the basement membrane zone. What is the most probably diagnosis ?

A. Dermatitis herpetiformis

B. Pemphigus vulgaris
C. Pemphigus foliaceus

D. Bullous pemphigoid

E. Varicella zoster infection

TEST 2

14. Choose the wrong answer about Henoch Schönlein purpura (HSP)

A. It is an acute immunoglobulin A (IgA) vasculitis

B. It is characterized by a generalized vasculitis involving the small vessels of the skin, the gastrointestinal
tract, the kidneys, etc.

C. Overall prognosis is good in most patient

D. Usually thrombocytopenia is presented

15. Cutaneous adverse effects from long-term systemic steroids may not include:

A. bacterial infections

B. skin thinning, purpura

C. acne

D. malnutrition

16. Choose the wrong answer:

A. Angioedema is an area of swelling of the lower lip of skin and tissue just under the skin of mucous
membranes
B. Edema could affect the gastrointestinal mucosa and leads to severe abdominal pain

C. ACE inhibitors can induce angioedema

D. C1-esterase inhibitors are the best in treatment allergic angioedema

17. Which treatment is not useful in urticaria:

A. Cetirizine

B. Topical glucocorticosteroids

C. Cyclosporine

D. Methylprednisolone

18. Which one of the answers below are not included in the Hanifin and Rajka criteria:

A. Pruritus

B. Dermatitis affecting flexural surfaces in adults and the face an extensors in infants

C. Chronic and recurrent course

D. Red dermographism

19. Choose the wrong answer:

A. Bullous lichen planus is rare

B. The plaques are crossed by fine white lines called Wickham striae

C. Oral prednisone is contraindicated in this condition


D. Sometimes lichen planus occurs along the autoimmune disorders

20. Which features are estimated in PASI score?

A. Erythema, induration

B. Erythema, induration, scale (also, % body surface area)

C. Erythema, scale

D. Erythema, quantity of plaques, scale

21. In lichen planus we can find:

1) Koebner phenomenon

2) Auspitz’s sign

3) Wickham striae

4) Nikolsky sign I

A. 1,2

B. 2,3

C. 3,4

D. 1,3

22. We don’t have cicatricial alopecia in:

A. DLE
B. Classic lichen planopilaris

C. Alopecia areata

D. Frontal fibrosing alopecia

23. Mark the incorrect answers concerning lichen planus:

A. pterygium is the most specific nail abnormality

B. buccal mucosa involvement is the most common in oral lichen planus

C. acitretin is the first line treatment

D. systemic corticosteroids are contraindicated

24. In the major criteria for atopic dermatitis (Hanifin and Rajka) we have:

A. pruritis

B. typical location: face and extensor surfaces in infants, flexural surfaces in adults

C. personal or family history of cutaneous or respiratory allergy

D. all answers are correct

25. Lichenification:

A. is a superinfection

B. is a thickening of the epidermis and accentuation of natural skin lines

C. is mostly located on elbows and knees in patients with atopic dermatitis


D. is treated with topical antibiotics

26. Choose the correct answer about lichen planus:

A) Lesions can appear on the skin or inside the mouth

B) The primary lesion is pustule

C) Can cause cicatricial alopecia

D) Long lasting erosive lichen planus can result in squamous cell carcinoma

1. A,B,C,D

2. A,C,D

3. A,C

4. A,B,C

27. Which answer is not connected with the side effects of cyclosporine:

A. Hypertension

B. Hypoglycemia

C. Renal failure

D. Nausea

28. Which drug would you recommend to a patient with hypertension and severe psoriasis?
A. Cyclosporine

B. Methotrexate

C. Prednisone

D. Cyclophosphamide

29 …… ?

A. Papule

B. Vesicle

C. Pustule

D. Erosion

30. PASI score in severe psoriasis is:

A. >5

B. >10

C. >20

D. >30

31. Raynaud’s phenomenon can be found in:

A. systemic sclerosis

B. systemic lupus erythematosus


C. dermatomyositis

D. all answers are correct

32. In drug-induced systemic lupus erythematosus we have antibodies:

A. anti-dsDNA

B. anti-Sm

C. anti-Ro

D. anti-histone

33. In the SLE criteria we don’t have:

A. Raynaud’s phenomenon

B. arthritis

C. serositis

D. thrombocytopenia <100,000

34. Risk factors for melanoma development:

A. phototypes I and II

B. Over 100 nevi

C. both answers are correct

D. none answer is correct


35. Malpighian stratum contains:

1. Stratum basale

2. Stratum spinosum

3. Stratum granulosum

4. Stratum corneum

A. 1,2

B. 2,3

C. 4

D. 3,4

36. Correct answers about erysipelas are:

1. The disease is caused by streptococcus pyogenes

2. Erysipelas can be recurrent

3. The typical location is lower leg (shank)

4. We treat it with oral steroids

5. Erysipelas never starts with fever

A) 1,2,3

B) 2,3,4,5
C) 2,3,4

D) 1,4,5

37. The primary lesion in ecythema is:

A. ulceration

B. papule

C. plaque

D. blister/pustule

38. Erythrasma is caused by

A. Staphylococcus aureus

B. Streptococcus pyogenes

C. Corynebacterium minutissimum

D. Pseudomonas aeruginosa

39. To treat the patient suffering from erysipelas you choose as the first line treatment:

A. Amoxicillin with clavulanic acid p.o.

B. Cefuroxime s.c.

C. Ichtillium p.o.

D. Tetracycline i.v.
40. Correct answers about pemphigus are:

1. The antibodies in pemphigus vulgaris are IgG

2. We observe mucosal involvement in pemphigus foliaceus

3. We start the treatment with oral antibiotics

4. Patient should be treated for at least 2 years

A. 1,3

B. 3,4

C. 1,2,4

D. 1,4

41. Nikolsky's sign is presented in:

A. Pemphigus vulgaris

B. Generalized pustular psoriasis

C. Lichen planus

D. A + B are correct

42. Bullous Pemphigoid:

A. Can be triggered by malignancies


B. Antibodies mostly depend on IgA class

C. The blister is flabby

D. The main onset of the disease is during infancy

43. The topical treatment in bullous pemphigoid you start with:

A. Clobetasol

B. Tacrolimus

C. Mupirocin

D. Cyclosporine

44. In SLE you can observe:

1. Leukopenia

2. Lymphocytosis

3. Anemia

4. Thrombosis

A. 1,3

B. 1,2,3

C. 2,3,4

D. 3,4
45. Scl-70 ANA is characteristic for:

A. SLE

B. Systemic sclerosis

C. Mixed connective tissue disease

D. DLE

TEST 3

46. Most cases of erythema multiforme are related to an infection caused by

A. CMV

B. HSV

C. VZV

D. S.pyogenes

E. S.aureus

47. Severe acne vulgaris can not cause:

A. atrophic ulcers

B. Keloids

C. Hyperpigmentation

D. Rhinophyma
E. Depression

48. Turn over time-migration of cells form basal layer to stratum corneum within a healthy epidermis
takes:

A. 24 hours

B. 1 week

C. 14 days

D. 28 days

49. Erythema nodosum can be due to:

A. Birth control pills

B. Tuberculosis

C. Sarcoidosis

D. Streptococcal infection

E. All of the above

50. Erythema nodosum is an example of:

A. Erythroderma

B. Big vessel inflammation

C. Urticaria

D. Panniculitis
E. Pathergy phenomenon

51. Factors which worsens rosacea include:

A. Sun exposure

B. Alcohol consumption

C. Hot beverages

D. All of the above

E. None of the above

52. The koebner phenomenon may be seen in:

A. Psoriasis

B. Erythema nodosum

C. Herpes Zoster

D. Linear scleroderma

E. All of the above

53. Which type of allergic mechanism is involved in the pathogenesis of contact dermatitis?

A. Type 1

B. Type 2

C. Type 3
D. Type 4

E. Type 5

54. Systemic treatment for psoriasis are following, except:

A. PUVA

B. Methotrexate

C. Oral glucocorticosteroids

D. Oral retinoids

E. Cyclosporine A

55. Target like lesion is characteristic for:

A. Discoid lupus erythematosus

B. Lichen planus

C. Scleroderma

D. Mycosis fungoides

E Erythema multiforme

56. Which immune complexes are commonly seen in Henoch-Schonlein purpura?

A. IgA

B. IgE
C. IgG

D. IgM

E. None of the above

57. Presence of cutaneous drug eruption are following except:

A. Erythema multiforme

B. Lichen planus

C. Atopic dermatitis

D. TEN

E. Urticaria

58. The primary skin lesion in lichen planus is:

A. Wheal

B. Papule

C. Pustule

D. Macule

E. Erosion

59. Which disease is strongly associated with diabetes mellitus?

A. Psoriatic arthritis
B. Atopic dermatitis

C. Seborrheic dermatitis

D. Necrobiosis lipoidica

E. Steven Johnson syndrome

60. Indicate the treatment for scabies

A. Topical retinoid

B. Erythromycin

C. Permethrin

D. Benzoyl peroxide

E. Azelaic acid

61. Wickham striae is typical for:

A. Eczema

B. Lichen planus

C. Mycosis fungoides

D. Sezary syndrome

E. Morphea

62. In the treatment of vitiligo one can use:


A. Topical tacrolimus

B. 5-FU topically

C. Terbinafine topically

D. Neomycin topically

E. Doxycycline orally

63. The gold standard of diagnosis of autoimmune bullous disease is:

A. Histopathology examination

B. Tzanck smear

C. Direct immunofluorescence

D. Indirect immunofluorescence

E. Dermoscopy

64. Which of the following methods should not be used in treating pemphigoid:

A. Systemic corticosteroids

B. Topical corticosteroids

C. Antimalarials

D. Azathioprine

E. PUVA
65. What is the first line management for dermatitis herpetiformis?

A. Prednisone

B. Dapsone

C. Gluten-free-diet

D. A+C

E. None of the above

66. The most useful diagnostic method in erythrasma is:

A. Culture

B. KOH

C. Dermoscopy

D. Wood’s lamp

E. Histopathology

67. What is typical dermatomyositis?

A. dsDNA antibody

B. Ro antibody

C. Malar rash

D. Scl-70 antibody

E. Gottron’s sign
68. Choose the wrong information about lichen planus:

A. It causes typical nail changes

B. Causes erosions on genital mucosa

C. Pruritus is very frequent symptom

D. Affects also hair follicles

E. Typical lesions are macules

69. Herald patch is typical for:

A. Fixed erythema

B. Pityriasis rosea

C. Pityriasis versicolor

D. Stevens Johnson

E. Rosacea

70. Digitate dermatosis is a type of

A. Psoriasis

B. Plaque parapsoriasis

C. Mycosis fungoides

D. Lamellar ichthyosis
E. Pemphigus vulgaris

71. To make a diagnosis of systemic lupus erythematosus according to SLICC 2012:

A. ≥4 criteria (at least 1 clinical and 1 immunologic) are required

B. >4 criteria (at least 1 clinical and 1 immunologic) are required

C. ≥1 criteria from either clinical or immunologic criteria is required

D. Non criteria is needed if there is a biopsy-proven cutaneous lupus

E. A and D are correct

72. Which of the skin lesions should be removed because of their possible malignancy (only
malignancies in one line):

A. nevus spilus, pyoderma granuloma, venous lake, cherry angioma

B. basal cell carcinoma, melanoma, spider angioma, angiokeratoma

C. Basal cell carcinoma, actinic keratosis, merkel cell carcinoma, fibrosarcoma

D. Melanoma, keratoacanthoma, basal cell carcinoma, skin tag

E. cutaneous horn, squamous cell carcinoma, basal cell carcinoma, halo-nevus

73. Sezary syndrome, which is correct ?

A. It is a cutaneous B cell lymphoma

B. Peripheral lymphadenopathy is present

C. Sezary cell can be found only in the skin


D. Rarely it can be erythrodermic

E. There is a diffuse hair loss and hyperkeratosis of palms and soles

74. Which can be treated using ciclopirox or terbinafine?

A. Impetigo

B. Tinea unguium

C. Condyloma acuminatum

D. Erythrasma

E. Genital candidiasis

TEST 4

75. In atopic dermatitis one of the postulated pathomechanisms is defect in:

A. transglutaminase

B. cornetin

C. filaggrin

D. PAF

76. Diagnosis of atopic dermatitis is based on criteria:

A. Cooka and Cook

B. Hanifin and Rajka


C. Hauley and Smith

D. Wiliams and Falco

77. Complication in atopic dermatitis may be

A. superinfection with S.epidermidis (Staph. Aureus)

B. eczema herpeticum

C. decreased transdermal water loss (increased)

D. all of the above

78. Phototherapy is not used in the treatment of:

A. atopic dermatitis

B. systemic lupus erythematosus

C. mycosis fungoides

D. psoriasis

79. Angioedema may be dependent with disturbances in complement system. What kind?

A. C1 inhibitor deficiency

B. decreased level of C5 and C6

C. overexpression of C4

D. increased activity of C2 protease


80. Which one is not a trigger factor for cholinergic urticaria?

A. exercise

B. overheating

C. cold water

D. stress

81. Acrodermatitis chronica atrophicans is:

A. I stage of Borreliosis

B. II stage of Borreliosis

C. III stage of Borreliosis

D. II stage of Leprosy

82. Mycosis fungoides is:

A. T cell lymphoma

B. B cell lymphoma

C. secondary skin lymphoma

D. special variant of BCC

83. Which one is correct?


A. patch stage of mycosis fungoides has good response to topical steroids

B. plaque stage is exacerbated by UV exposure

C. tumor stage is paraneoplastic syndrome

D. plaque stage is a II stage of mycosis fungoides

84. The most common indication for Q-switched laser is:

A. treatment of vascular lesions

B. removing tattoos

C. removing scars

D. face rejuvenation

85. AGEP is not characterized by:

A. high fever

B. numerous big follicular, sterile pustules

C. burning or pruritis

D. lesions beginning on the face or in the major intertriginous zones

86. Which factor is not used in SCORTEN:

A. age

B. serum glucose level


C. serum bicarbonate level

D. blood pressure

87. Wickham’s striae is characteristic for:

A. lichen planus

B. psoriasis

C. contact dermatitis

D. sarcoidosis

88. Koebner phenomenon may occur in the course:

A. herpes zoster

B. pemphigus vulgaris

C. alopecia areata

D. lichen planus

89. Systemic retinoids are not used in the treatment of:

A. lichen planus

B. atopic dermatitis

C. psoriasis

D. cutaneous lymphoma
90. Which skin lesion may be primary and secondary?

A. vesicle

B. pustule

C. wheal

D. nodule

91. Lichenification is:

A. thickening of the epidermis and accentuation of natural skin lines

B. fibrous tissue replacement of the skin defect or after inflammation

C. secondary bacterial infection

D. accumulation of stratum corneum

92. Naevus coeruleus is:

A. a solitary nodule

B. always developing into melanoma

C. due of blue gray

D. all of the above are true

93. In treatment of metastatic melanoma we can use


A. ipilimumab

B. vismodegib

C. omalizumab

D. adalimumab

94. To grade melanoma in histopathology we use

A. Auspitz scale

B. Breslow scale

C. Cowden scale

D. Darter scale

95. Pyoderma gangrenosum:

A. is infectious neutrophilic dermatosis (non-infectious)

B. ulcerative variant is the most common

C. is never associated with systemic conditions

D. the first-line treatment is cephalosporin II generation

96. In psoriasis histopathology we can observe:

A. hyperkeratosis

B. pustules of Kogoj
C. microabscesses of Munro

D. all of the above

97. CASPAR criteria are important to diagnose:

A. lupus

B. psoriasis arthritis

C. systemic sclerosis

D. psoriasis

98. SAPHO syndrome is associated with:

A. pustulosis

B. rosacea

C. hidradenitis suppurativa

D. orofacial granulomatosis

99. Which drug is not used in treatment of Raynaud phenomenon:

A. sildenafil

B. alprostadil

C. propranolol

D. nifedipine
100. Cutaneous adverse effects from long term systemic steroids may not include:

A. Bacterial infection

B. Skin thinning, purpura

C. Acne

D. malnutrition

101. Choose the wrong answer:

A. Bullous lichen planus is rare

B. The plaques are correct by fine white lines called Wickham striae

C. Oral prednisone is contraindicated in this condition

D. Sometimes lichen planus occurs along with autoimmune disorders

102. Which features are estimated in PASI score?

A. erythema, induration

B. erythema, induration, scale

C. erythema, scale

D. erythema, quantity of plaques, scale

103. In lichen planus we can find:


1) Koebner phenomenon

2) Auspitz’s sign ( for psoriasis)

3) Wickham’s striae

4) Nikolsky’s sign I

A. 1,2

B. 2,3

C. 3,4

4. 1,3

104. Mark the incorrect answer concerning lichen planus:

A. pterygium is the most specific nail abnormality

B. buccal mucosa involvement is the most common in oral lichen planus

C. acitretin is the first line treatment

D. systemic corticosteroids are contraindicated

105. Lichenification:

A. is a superinfection

B. is a thickening of the epidermis and accentuation of natural skin lines

C. is mostly located on elbows and knees in patients with atopic dermatitis


D. is treated with topical antibiotics

107. Choose the correct answer about lichen planus:

A) lesions can appear on the skin or inside the mouth

B) the primary lesion is pustule

C) can cause cicatricial alopecia

D) long lasting erosive lichen planus can result in squamous cell carcinoma

1. A,B,C,D

2. A,C,D

3. A,C

4. A,B,C

108. Which answer is not connected with the side effects of cyclosporine:

A. Hypertension

B. Hypoglycemia

C. Renal failure

D. Nausea

109. Which drug would you recommend to a patient with hypertension and severe psoriasis

A. Cyclosporine
B. Methotrexate

C. Prednisone

D. Cyclophosphamide

110. The primary lesion in herpes zoster is:

A. papule

B. Vesicle

C. Pustule

D. Erosion

111. PASI score in severe psoriasis is:

A. >5

B. >10

C. >20

D. >30

112. Raynaud’s phenomenon can be found in:

A. systemic sclerosis

B. systemic lupus erythematosus

C. dermatomyositis
D. all answers are correct

113. Risk factors for melanoma development:

A. phenotypes I and II

B. Over 100 nevi

C. both answers are correct

D. none answer is correct

114. Malpighian stratum contains:

1. Stratum basale

2. Stratum spinosum

3. Stratum granulosum

4. Stratum corneum

A. 1,2

B. 2,3

C. 4

D. 3,4

115. Correct answers about erysipelas are:


1. The disease is caused by Streptococcus Pyogenes

2. Erysipelas can be recurrent

3. The typical location is lower leg (shank)

4. We treat it with oral steroids

5. Erysipelas never starts with fever

A. 1,2,3

B. 2,3,4,5

C. 2,3,4

D. 1,4,5

116. The primary lesion in ecthyma is:

A. ulceration

B. papule

C. plaque

D. blister/pastule

117. Erythrasma is caused by

A. Staphylococcus aureus

B. Streptococcus pyogenes
C. Corynebacterium minutissimum

D. Pseudomonas aeruginosa

118. To treat the patient suffering from erysipelas you choose as a first line treatment:

A. Amoxicillin with clavulanic acid p.o.

B. Cefuroxime s.c.

C. Ichtilium p.o.

D. Tetracycline i.v.

139. In atopic dermatitis one of the postulated pathomechanisms is defect in:

A. transglutaminase

B. cornetin

C. filaggrin

D. PAF

140. Diagnosis of atopic dermatitis is based on criteria:

A. Cooka and Cook

B. Hanifin and Rajka

C. Hauley and Smith

D. Williams and Falco


141. Complication in atopic dermatitis may be

A. superinfection with S.epidermidis (Staph. Aureus)

B. eczema herpeticum

C. decreased transdermal water loss (increased)

D. all of the above

142. Phototherapy is not used in the treatment of:

A. atopic dermatitis

B. systemic lupus erythematosus

C. mycosis fungoides

D. psoriasis

TEST 5

143. Which of the following lesion usually leads to scarring:

A. Lichenification

B. Scaling

C. Wheal

D. Macule

E. None of the above


144. Acantosis nigricans

A. It is a paraneoplastic disease

B. It is a fungal infection caused by Dermatophytes nigricans

C. It is a bacterial infection caused by Corynebacterium nigricans

D. It is a variant of solar keratosis

E. None of the above

145. Allergy to chromium is related to

A. Type I allergic mechanism

B. Type II allergic mechanism

C. Type III allergic mechanism

D. Type IV allergic mechanism

146. Pustules:

A. can be observed in some variants of psoriasis

B. can be seen in inflammatory acne

C. are typical lesions for folliculitis

D. all answers are correct


147. Hemidesmosomes are:

A. located between epithelial cells

B. connecting epithelial cells with basal membrane

C. are part of the endosome system of dermal phagocytes

D. are endosomes of sebocytes

149. Paraneoplastic diseases are the following, EXCEPT:

A. Erythema multiforme

B. Erythema gyrathum repens

C. Necrolytic migratory erythema

D. Erythema annulare centrifugum

150. Viral diseases are following, EXCEPT:

A. Plane warts

B. Plantar warts

C. Common warts

D. Seborrhoeic warts

151. An example of an autoimmune blistering disease is:

A. Dermatitis herpetiformis
B. SSSS

C. Morphea generalisata

D. Impetigo bullosa

152. High risk melanoma is related to:

A. Type IV skin phototype

B. Chronic sun exposure

C. Sunburns in childhood

D. HPV infection of melanocytes

153. The most important role in the protection against transepidermal water loss is played by:

A. Basal layer

B. Spinal layer

C. Granular layer

D. Cornified layer

154. Melanocytes are located:

A. between basal cells

B. between spinal cells

C. just below basal membrane zone


D. just below granular layer

155. Mast cells are:

A. residual in the epidermis

B. playing major role in type II allergic reactions

C. are equipped with high number of high affinity IgE receptor (FceRI)

D. playing the major role in pathogenesis autoimmune connective tissue diseases

156. Wheal is characteristic for

A. atopic dermatitis

B. urticaria

C. guttate psoriasis

D. erythrasma

157. Pemphigus vulgaris

A. is a mild disease of pilosebaceous unit

B. is a bacterial disease

C. typically pustules are present

D. all of the above are correct

E. all of the above are wrong


158. Systemic isotretinoin

A. is used in the treatment of dermatomyositis

B. is highly teratogenic

C. is used for the treatment of melanoma

D. is never used as systemic agent

159. The true statement of atopic dermatitis is:

A. The percentage of affected people has decreased in the last 40 years

B. It affects higher rate of population in developing countries when compared to western countries

C. The prevalence of atopic dermatitis is higher in children than in adults

D. It rarely affects asthmatic patients

161. Indicate bacterial diseases

A. Erysipelas

B. Erythrasma

C. Pityriasis versicolor

D. molluscum contagiosum

162. For the treatment of acne one should NOT use:


A. Cyclosporine A

B. Oral antibiotics

C. Topical antibiotics

D. Topical retinoids

2012 FINAL

vitiligo.
neonatal lupus eythematous.

- Mother Patch (Herald patch) found in which? A. Lichen planus B. Pityriasis ..... C. Pityrisia
rosea Gilbert

- Koebner phenomenon in which? Lichen planus, other options don't

- Nikolsky sign not observed in? Lichen planus, other options can be observed

- Seborrheic dermatitis treatment

- Which have dome-shaped lesion with bla bla...(dont remember, please continue). Options :
Lichen planus, Zoster, Psoriasis

- Limited only to the skin? Lichen planus, Dermatomyositis, etc

- CI with phototherapy/PUVA? (Options with 3 diseases together given).. eg A. SLE, Psoriasis,


LP B. SLE, Dermatomyositis, Rosacea
- Rosacea treatment

- Borelliosis treatment : Antiviral 5 days or 21 days, Antibacterial 5 days or 21 days

True about dermatitis herpetiformis.

Layers of skin. Corneum, granuloma, lucidum, basal layer.

Similarities between paoriasis and syphilis stage two.

Syphilis primary lesion.

Where are antibodies in pemphigus vulgaris?? Or some other disease.

Treatment of rosacia.
laser therapy(which one is not correct pairing)
primary syphilis..DD same with SSC
wicham striae
side effect of tropical steroid
secondary lue recens n psoariasi=cutaneous lesion

Morbus bowen is:

Treatment of Acne vulgaris: I think Benzoyl Peroxide + clindamycin, but topical retinoids may
also work.

Side effects of steroids: atrophy, telangiectasis, striae, ______, (A: all of the above)

Rhinophyma is hyperplasia of sebaceous glands on the nose.


Desmoglein 3 in pemphigus vulgaris

What does re-PUVA mean: retinoin + PUVA

Which type of psoriasis is most common: chronic plaque.

Which of these do not have Nikolsky sign:

2010 FINAL

1. Suitable treatment for comedones acne is:

A. Topical retinoids

B. Oral antibiotics

C. Oral antibiotics plus topical retinoid or benzoyl peroxide

D. Topical calcineurin inhibitors

E. lasers

2. For inflammatory acne (papulo-pustular) one can use:

A. Topical gentamicin

B. Systemic gentamicin

C. Oral retinoid plus oral antibiotic

D. Alcohol based washed solutions plus non steroidal anti-inflammatory drugs


E. Topical or systemic antibiotic plus topical retinoids

3. Scabies

A. Treatment of topically applied 5% permethrin is usually effective

B. Itching is usually mild (severe)

C. Infants are never affected

D. Erosions on mucous membranes can be present

E. something something something arms and buttocks are predominantly affected

4. The most common contact sensitizer in Western countries is

A. Nickel
B. Dust mite
C. Birch pollen
D. Grass pollen
E. Milk

5. Discoid lupus erythematosus:

A. Hypopigmentation can be present


B. Usually occurs on buttocks
C. Follicular parakeratosis can be present
D. Renal involvement occurs in 50 % of patients
E. Ro/La antibodies are usually present

6. SCLE can mimic:

A. Psoriasis
B. erythema nodosum
C. Pyoderma gangrenosum
D. Morphea
E. Lichen planus

7. DRESS is acronym of:

A. Drug rash, eosinophilia, systemic symptoms


B. Dermatitis, redness, esophagus, sclerosis, sclerodactyly
C. Drug induced redness, elevated enzymes, scalded skin
D. Deafness, retinopathy, encephalopathy, skin symptoms
E. Deafness, retinopathy, ECG, skin symptoms

8. Which is the most frequent type of melanoma:

A. nodular
B. lentigo maligna melanoma
C. Superficial spreading
D. Amelanotic
E. Acral lentiginous melanoma

9. This type of acne occurs in teenage boys. It is a severe cystic acne with suppuration and ulceration
and is characterized by acute onset of fever, malaise, generalized arthralgia and leukocytosis. The
description fits for:

A. Acne conglobata
B. Acne excoriee
C. Tropical acne
D. Chloracne
E. Acne fulminans

10. Erythema multiforme syndrome can be triggered by:

A. Herpes simplex infection


B. Streptococcus pyogenes infection
C. drugs such as sulfonamides, penicillin
D. idiopathic
E. All of them

11. Alopecia can be present in:

A. SLE, DLE, Leus


B. lichen planus, LABD, erythema multiforme
C. fixed erythema, psoriasis, impetigo
D. SSSS, TEN, DRESS
E. Nummular eczema, PLEVA, Pityriasis rosea

12. Choose the most suitable approach to the treatment of typical seborrhoeic warts (seborrhoeic
keratosis):

A. Curettage
B. Total excision
C. They do not require treatment
D. A and C are correct
E. Radiotherapy

13. Chronic urticaria:

A. Aspirin urticaria is the most common variant


B. A single wheal lasts more than 6 weeks
C. Topical steroids are the first line treatment
D. No one cares!!

14. Pyoderma gangrenosum:

A. Pseudomonas aeruginosa is a typical causative bacteria


B. surgical treatment associated with antibiotics should be recommended
C. Dapsone can be helpful
D. Affects patients with immunodeficiency
E. B and D answers are correct

15. Major criteria to establish the diagnosis of atopic dermatitis are:

A. Pruritus, typical distribution of eczematous lesions, chronic and relapsing course,history of


atopy in patient and/or his family
B. Skin dryness, flexors involvement, pruritus, excoriations
C. elevated serum IgE, skin dryness, flexors involvement, chronic and relapsing course
D. early onset, pruritus, flexors involvement, elevated serum IgE
E. skin dryness, flexors involvement, pruritus, history of atopy in patients and/or his family

16. The most common factors that provoke acute urticaria are:

A. Food and drugs


B. Cold
C. Pressure
D. Infections
E. Haptens

17. Vitiligo is:

A. An autoimmune disorder
B. viral disease
C. of bacterial etiology
D. an adverse drug reaction
E. autosomal recessive disease

18. The most common nevi in newborn are:

A. Vascular
B. Epidermal
C. Melanocytic
D. Seborrheic
E. Other

19. The proper procedure for rapidly horizontally growing atypical melanocytic nevus on face is:
A. Total excision of the lesion followed by histopathological evaluation
B. a punch biopsy performed as soon as possible followed by histopathological evaluation and laser
therapy
C. a shave biopsy performed as soon as possible followed by histopathological evaluation and
cryotherapy
D. Sun protection of this lesion with sunscreen SPF 50+ and a clinical evaluation every 12 months
E. topical cytostatic treatment for 2 weeks and punch biopsy with histopathological evaluation plus
radiotherapy of local lymph nodes

20. Invasive squamous cell carcinoma:

A. usually develops on pre-existing actinic keratosis


B. lymph node metastases are present in about 5 % of patients
C. treatment is based on surgical excision with histologic control of margin
D. is frequently associated with p53 mutations
E. All of the above answers are correct

21. Treatment of actinic keratosis Do NOT include:

A. imiquimod
B. topical 5-fluorouracil
C. topical diclofenac in hyaluronic acid
D. Mohs micrographic surgery
E. Cryotherapy

22. Fixed erythema:

A. the most severe adverse reaction to drugs


B. systemic corticosteroids is treatment of choice
C. variant of erythema multiforme major
D. males are more frequently affected
E. reoccurs at the same site after exposure to provoking
23. Erythema nodosum:

A. can be preceded by streptococcal infections


B. can be a sign of sarcoidosis (Lowgren’s syndrome)
C. can be caused by oral contraceptives
D. B and C answers are correct
E. A, B, and C are correct

24. Allergic contact dermatitis is:

A. IgE mediated disease


B. irritant reaction to detergents
C. Response to T-cell haptens
D. reaction to protein allergen
E. due to the deranged to endothelial cells by immune

25. Wheals lasting for 1-3 days are characteristic for:

A. cholinergic urticaria

B. allergic urticaria

C. Autoimmune urticaria

D. urticaria vasculitis

E. cold urticaria

26. Choose the wrong answer. Rosacea:

A. affects about 10% of fair-skinned people

B. occurs predominantly in women

C. typical skin lesion is a comedone


D. can be treated with oral antibiotics

E. can be treated with topical metronidazole

27. Seborrheic dermatitis:

A. Is more common in males


B. Is caused by Trichophyton rubrum
C. The typical primary lesion is a pustule
D. Affects only adults
E. Can be diagnosed with patch testing

28. Which of the following antibodies DOES NOT occur in SLE:

A. Ro/La

B. RNP

C. dsDNA

D. BP180

E. Sm

29. In morphea one NEVER observe:

A. Lilac ring

B. presence of autoantibodies

C. face involvement

D. Raynaud phenomenon
E. lesion of alopecia

30. NOT TYPICAL for dermatomyositis is:

A. Heliotrope

B. V-sign

C. Koebner sign

D. Gottron sign

E. Elevated liver enzymes (AST and ALT)

31. Typical localization for seborrheic dermatitis are all except:

A. Eyebrows

B. Nasolabial folds

C. Lower eyelids

D. Scalp

E. Sternal Area

32. Collarette desquamation is characteristic for:

A. psoriasis

B. pityriasis rosea

C. seborrheic dermatitis
D. pemphigus

E. atopic dermatitis

33. Rhinophyma is:

A. superficial bacterial infection of groins caused by Corynebacterium minutissimum

B. Nose changes in late lues

C. Variant of rosacea

D. Kaposi sarcoma lesion on face in AIDS patients

E. none of the above

34. Which antinuclear antibody is NOT a marker of the disease:

A. SLE-dsDNA
B. CREST-ACA
C. diffuse scleroderma-Scl 70
D. Dermatomyositis-Sm
E. SCLE-Ro

35. Indicate the symptom which is NOT present in skin lesions of DLE:

A. Erythema
B. Infiltration
C. Follicular hyperkeratosis
D. Scarring
E. purpura

36. What is true about actinic keratoses (AK):


A. AK are common and represent an early stage in the development of basal cell carcinoma
B. AK are mainly associated with tobacco smoking
C. AK should be treated surgically
D. AK develop mainly on the trunk
E. AK serve as a marker of excess sun exposure

37. What is NOT characteristic for:

A. CREST variant-sclerodactyly
B. limited scleroderma-central cutaneous involvement
C. diffuse scleroderma-hyper and hypopigmentation
D. CREST variant-presence of anticentromere antibodies
E. all forms of SSc-presence of Raynaud's phenomenon

38. The most common variety of basal cell carcinoma (BCC) is:

A. superficial BCC
B. nodular BCC
C. morpheaform BCC
D. pigmented BCC
E. ulcerative BCC

39. Squamous cell carcinoma:

A. is the most common skin malignancy


B. do no metastasize
C. develops more frequently in immunosuppressed patients
D. frequently is associated with malignant melanoma
E. mainly develops in females aged.

40. Which photodermatosis occurs the most frequently:

A. Polymorphic light eruptions


B. solar urticaria
C. xeroderma pigmentosum
D. porphyria cutan
E. Photoallergy

41. Which statement referring to sunscreen is FALSE:

A. contain UV filters
B. protect skin from UV
C. should be applied once daily
D. sometimes produce photoallergy reactions
E. should be used in PLE

42. Xeroderma pigmentosum:

A. is characterized by hypersensitivity to visible light


B. occurs in elderly
C. belongs to connective tissue disease
D. starts in infancy childhood
E. is mild photodermatosis

43. Phototoxicity/photoallergy is NOT observed after therapy with:

A. sulfonamides
B. tetracyclines
C. penicillins
D. Phenothiazines
E. antiarrhythmic drugs

44. Pemphigus vulgaris is characterized by:

A. presence of anti-desmoglein-1 antibody


B. suprabasal acantholysis
C. late onset
D. good response to low dose of corticosteroids
E. Koebner phenomenon

45. Nikolsky sign is characteristic for:


A. Toxic epidermal necrolysis
B. lichen planus
C. pemphigoid
D. psoriasis
E. erythema nodosum

46. Papule in lichen planus is NOT characterized by:

A. purple coloration
B. polygonal shape
C. pruritus
D. can be erosive
E. ulceration and cicatrisation

47. Lichen planus can affect:

A. only skin
B. skin and mucous membranes
C. skin and nails
D. skin and scalp
E. skin, and mucous membranes, scalp and nails

48. The diagnosis of Purpura Schonlein-Henoch is based on:

A. thrombocytopenia
B. Normal ESR
C. No general symptoms
D. IgA deposited in the skin (direct immunofluorescence)
E. Normal urine test

49. Not found in typical biopsy of psoriatic pattern:

A. Acanthosis
B. Acantholysis
C. Parakeratosis
D. Hypogranulosis
E. Neutrophils high in dermis

50. Commonly seen in psoriasis:

A. Alopecia
B. Christmas-tree distribution
C. Collarette of scale
D. Deep red color
E. Mucous membrane involvement

51. How do you differentiate scalp psoriasis and seborrheic dermatitis?

A. Presence of alopecia
B. Ear and face involvement
C. Presence of scaling
D. Presence of itch

52. PUVA not used in:

A. Lichen planus
B. Atopic dermatitis
C. Psoriasis
D. Vitiligo
E. Scleroderma
F. Parapsoriasis

53. Systemic treatment in psoriasis:

A. Always
B. When over 20% is affected
C. In pregnancy
D. Commonly tacrolimus (Topical)

54. Which is not a contraindicated of PUVA:


A. Epilepsy
B. Cardiomyopathy
C. Hepatitis
D. Children
E. Pregnancy

55. Erythema multiforme affects predominantly:

A. Distal extremities

B. Flexors surfaces

C. Upper back

D. Groins and axilla

E. Abdomen

56. Melanoma can evolve from:

A. Normally looking skin

B. Common melanocytic nevus

C. Atypical (dysplastic) melanocytic nevus

D. A and C answers are true

E. All of the answers are correct

57. Breslow scale is used to describe

A. skin phototype

B. the result of a path test


C. histopathological infiltrate of melanoma

D. weal/erythema ratio in prick testing

E. the level of skin sclerosis in systemic sclerosis

58. Morbus Bowen

A. can be treated with cryotherapy

B. is a variant of lentigo

C. is located on mucosal membranes

D. is primarily malignant

E. telangiectasias are present typically in large lesions

59. Mycosis fungoides

A. Has worse prognosis than Sezary Syndrome

B. Chronic sun exposure is a risk factor for MF

C. Cytostatic drugs are the first line therapy for the erythematosus stage

D. Retinoids can be helpful (bexarotene)

E. Is never erythrodermic

60. Which clinical feature is the most characteristic for psoriatic arthritis:

A. asymmetry of joints involvement


B. symmetry of joints involvement

C. absence of skin changes


D. None of the above

61. Poikiloderma, not true:

A. May be congenital
B. Atrophy
C. Scaling
D. Hypopigmentation
E. Telangiectasia
F. Seen in dermatomyositis

62. Biopsy shows IgG against hemidesmosome proteins:

A. 20-40 year at onset (seen in elderly)


B. More common in males
C. Begins with pruritic urticarial papules
D. Mucous membrane lesions are uncommon
E. Serum IgG levels are prognostic

63. Allergic contact dermatitis, not correct:

A. Type 4 hypersensitivity
B. Clinical inflammation 5 -7days after first exposure
C. Rash may last 3w
D. Sensitized individual rash <48h
E. Diagnosed by patch test

64. Not a predisposing factor for ICD:

A. Atopy
B. Hobby
C. Wet-Work
D. Asthma
65. Pemphigus foliaceus, not true:

A. 5% of pts receiving d-penicillamine or captopril


B. May be endemic
C. Mucous membrane lesions
D. Autoantibodies against desmoglein 1
E. Bullas usually not seen

66. Subcutaneous inflammatory red/blue nodules that become confluent and create painful
subcutaneous plaques. And livedo reticularis. What is the underlying mechanism?

A. Hypersensitivity
B. Bacterial infection
C. Viral infection
D. Vasculitis
E. Idiopathic

67. Discoid LE, which is incorrect?

A. Central atrophy
B. Alopecia
C. Oral mucosa involvement
D. 50% spontaneously regress
E. 20% progress to SLE

68. Flushing, facial redness, telangiectasia, pustules, papules and rhinophyma: Choose appropriate
option for this disease:

A. May be treated with topical GCS


B. PUVA treatment may be used
C. Usually treated by antibiotics
D. Never seen in children

69. Dermatomyositis, which is incorrect:


A. May be seen in children
B. May have no muscle involvement
C. Mucous membrane involvement
D. Poikiloderma
E. Alopecia

70. Dermatomyositis, choose incorrect:

A. Look for underlying malignancy


B. Biopsy muscle
C. Biopsy skin
D. Normal electromyogram
E. Jo1 autoantibodies (anti Mi-2 antibodies as well)

71. Which of the following is not treatment of dermatomyositis?

A. Sun protection
B. Systemic GCS
C. Antimalarials
D. Antihistamines
E. Calcipotriol

72. Alopecia areata, find incorrect:

A. Usually adults
B. Defined hair loss
C. Nail changes
D. Pitting of skin
E. Spontaneous remission

73. Which is not a complication of chlamydia in males:

A. Epididymitis
B. Conjunctivitis
C. Prostatitis
D. Infertility

74. Not a complication of Neisseria in in males?

A. Tonsillitis
B. Reiter’s syndrome
C. Cowperitis
D. Periurethral abscess
E. Edinei

75. Which of the following is seen in Reiter’s syndrome?

A. Conjunctivitis
B. Hepatitis
C. Arthritis
D. Urethritis
E. All
F. A, B, D
G. B,C,D
H. A,C,D

76. Neisseria Gonorrhoeae:

A. Gram stain shows blue/purple intracellular cocci


B. Only 50% is seropositive
C. Can be cultured
D. Negative sugar fermentation test
E. Males usually asymptomatic

77. Causes an infection of glans penis:

A. Trachomatis
B. Candida albicans
C. NG
D. CT
E. HIV

78. First detectable protein in HIV is:

A. P31
B. P27
C. P24
D. P17

79. Found in HIV 1 but not HIV2:

A. Gp 105
B. Gp120
C. P31
D. P24
E. P17

80. Serological window:

A. Found in HIV, syphilis and NG


B. Presence of symptoms is absence of serological verification
C. Can be diagnosed by dark field microscopy
D. Negative unspecific tests, but positive specific tests

81. Non infectious syphilis :

A. Chancre
B. Condyloma lata
C. Pat with generalized rash, no genital ulcers
D. Paralysis progressive

82. Congenital and acquired do not have in common:

A. Condyloma lata
B. General lymphandenopathy
C. Alopecia
D. Papular rash

83. Congenital HIV:

A. Risk is 20% when mother Is treated


B. Detected by culture of lymphocytes
C. Detected by serology
D. Uncommon

84. Which localization IS NOT typical for pustular psoriasis?

A. palms

B. feet

C. trunk

D. scalp

E. groins

85. Which therapy is the most effective in pustular psoriasis?

A. methotrexate

B. retinoids

C. cyclosporine

D. PUVA

E. NB-UVB
2009 FINAL
1. Recklinghausen's disease
a) chert hands and feet
b) Soft fibroma
c) café au lait spots
d) nodules Koen

2. DLE and lupus tuberculosis should be differentiated, because in both scarring and ulceration
occurs

3. Ulcerative tuberculosis (primary inoculation TB/TB chancre):


a) in patients with organ tuberculosis
b) does not occur on the mucous membranes
c) there is a softening of the lymph nodes
d) soft base changes

4. Restrictions in autumn and spring are in


a) erythema induratum( either a or b)
b) Sarcoidosis
c) tuberkuloid prosówkowy
d) tuberkuloid guzkowo-gangrenous

5. Glucagonoma
a) diabetes
b) glossitis
c) lesions
d) anemia

6. PAN (either a or b)
a) thrombocytopenia
b) changes in pulmonary
c) polineuritis (mononeuritis)
d) urticaria vasculitis (no)

Livido reticularis yes

7. SCLE
a) scarring (no scaring)
b) anti-Ro
c) changes in psoriasiform
d) sensitivity to sunlight

8. Antibodies in scleroderma ANA +ve


a) Native DNA
b) topoisomerase -1 (diffuse)
c) centromeres (limited)
d) histones

9. SSCl
a) anti-Scl-70 (antitopo)diffuse
b) Raynaud's phenomenon diffuse n limited but raynaud lame kat limited
c) a slow process limited years raynaud br ade disease
d) there are no changes in organ morphea....localised Ssc

10. Dermatomyositis
a) changes in facial erythema
b) symptom Gotrona
c) smooth muscle involvement
d) calcium deposits on joints (calcification of muscle...deposit of ca in childern)

11. papules, pustules - satellites -> odp advance yeast (candida)


12. Streptococcal infection - Roza
13. Impetigo Contagious Diseases
a) more often in people with AIDS
b) treat locally
c) the infection spreads easily in
d) good general condition of patients

14. Hidradenitis suppurativa not occurs before puberty, because it relates to apocrine glands.

15. Least likely to use in acne vulgaris:


a) erythromycin
b) tetracycline
c) minocycline
d) doxycycline
e) clindamycin

16. acne vulgaris from roseola makes a difference:


a) lumps
b) spots
c) vasodilatation
d) blackheads

17. Granuloma naczyniakowy (vascular tumor) either c or d


a) neoplasia of the capillaries (no)
b) differentiate from MF (no)
c) bleeds easily
d) provoked by injuries

18. Blisters arise in places trauma in:


a) Authority
b) Epidermolysis bullosa hereditaria
c) PCT
d) The disease Duhring

19. Ordinary ichthyosis-dominant inheritance - had:


a) skin changed at birth no—3 months
b) free of bends and folds around
c) hyperkeratosis of the hands and feet ..no usually spare hand
d) coexists with AD

20. AD - for big manifestations should not be -> elevated IgE


21. AD -> prone to recurrent skin infections (2ndary infection)

22. Contact dermatitis to nickel:


a) more often in women
b) changes not only in the hands and feet
c) often ornaments
d) coexist sensitivity to silver

23. allergic to perfumes most common in the -> eczema cruris

24. Fungus pricking -> dark spots and uneven a split hairs

25. Permanent hair loss in


a) DLE
b) lichen planus atrophicus
c) Alopecia by retinoids
d) Tinea mikrozarodnikowa

26. after treatment with a msc neotigasonem contraception because it is carcinogenic

27. Fungi differentiate the groin:


a) erythrasma
b) exudative psoriasis
c) advance yeast
d) The prohibition Epidermophyton

28. Generally treat Candidiasis:


a) itraconazole
b) fluconazole
c) griseofulvin
d) terbinafine

29. Actinomycosis usually takes around mandibular because it takes the internal organs
30. Hard deskowaty infiltration -> actinomycosis
31. Lyme Disease
a) ECM
b) ACA

32. Lupus tuberculosis


a) systemic treatment
b) coincides with the organ tuberculosis
c) Ulcers
d) outstanding tuberculin reactions

33. Home plate

34. Athlete's foot cure 4 weeks, because it always takes the nail shafts

35. Changes in the mucosa are diagnostic for


a) candidiasis
b) lichen planus
c) DLE
d) Pemphigus vulgaris yes...foliceus not
36. androgenetic alopecia

37. Herpes simplex

38. Squamous cell carcinoma

39. Molluscum contagiosum -> evokes poxvirus

40. Molluscum contagiosum - mostly in people with HIV, treatment with curettage and imiqwimod

41. Keratoacanthoma benign lesion

42. Flat warts


a) more often in adults
b) lesions in EV
c) HPV 6
d) They may disappear spontaneously

43. Erythema fixum

44. DD for Eruptions in guttate psoriasis can be:


a) dandruff pink Gibert
b) pityriasis lichenoides chronica + lichen planus
c) parapsoriasis

45. Aspirin - if I type of reaction and whether leukotrienes

46. Serum sickness

47. Phototoxic reactions - from which depend


48. Differentiation of urticaria - interview (eryhthema multiforme n urticarial vasculitis

49. Where to use corticosteroids in liquid?

50. Hereditary angioedema

51. Urticatia vasculitis

52. PUVA treatment


a) Polymorphic light reaction

53. Radiodermatitis Chronika - it blooms?

54. Pemphigoid and pemhigoid neoplasticus

55. antibodies in pemhigus

56. gestational pemphigoid

57. Diagnosis morbus Duhring

58. panniculitis - check the pancreas and is associated with an inhibitor of alpha-antitrypsin
59. When there may be purpura
60. Leukoklastyczne vasculitis
61.Owrzodzenia leg - API, venous hypertension
62. Sarcoidosis
63. Seborrheic warts - the laser-trelata band may be stained
64. team signs of dysplastic
65. Actinic keratosis
66. Actinic chemicals - arsenic, mainly hands and feet
67. Sezary syndrome - fever not
68. BCC
69.poliosis
70. OCENYINFESTATION progressive - the result of spinal cerebrospinal fluid examination (wbc >5,
protein > 40%)
71. VLDR negative reaction, so what it might be
72. Early-keel which may occur syphilis alopecia
73. inflammation of the aorta in a patient before 40rz, suspect syphilis?
74. Why in the control reaction after treatment, we use FTA?
75. composition of the vaccine against cervical cancer
76. syndrome, fitz-hugh-curtis
77. identification of chlamydia - a direct immunofluorescence
78. bad cured gonorrhea - betalactamase or chlamydial nakadzenie
79. Genital warts
80. period of acute viremia in HIV

2019 FINAL - POLISH DIVISION

1-6 MISSING

7.

8. The correct statement for the treatment of acne vulgaris is:


A. General antibiotic treatment should not last longer than 2 weeks
B. General antibiotic treatment should not be used together with topical treatment
C. Isotretinoin acts as a teratogen
D. In Acne fulminans, treatment is oral corticosteroids

9. Rituximab is used in dermatology because:


A. Blocks the binding of IL-17 to the receptor on T-lymphocytes
B. Reduces TNF-alpha activity
C. Acts in a cytotoxic way against B lymphocytes CD20+
D. Reduces the activity of IL-2 producing cells
E. Inhibits the synthesis of IL-23

10. In a 60 year old female patient, there are many light brown papule and hyperkeratotic patches
that are located on the trunk. She had them for a couple of years. This nevi are most probably:
A. epithelial
B. sebaceous
C. pigmented
D. vascular
E. apocrine

11. Choose the correct statement about actinic keratosis:


A. Actinic keratosis occurs more often in women
B. Neoplastic transformation leads to the development of melanoma
C. Dermascope shows a pattern of red pseudo grid (?) (strawberries)
D. It is a point of focus/outbreak on a smooth surface
E. Photoprotection does not affect the development of changes

12. What is correct regarding melanoma


A. Phototype 4 and 5 are at risk
B. Always develops from a Nevus
C. Usually causes pain
D. The number of nevi does not correlate with the risk of melanoma
E. The BRAF mutation is a key factor in pathogenesis

13. Which disease is associated with oncogenetic infection and HPV types:
A. Genital warts
B. Buschke - Lowenstein tumor
C. Lichen sclerosus
D. Bowenoid papulosis
E. Laryngeal papillomatosis

14. Target lesions can be found in the following disease


A. Sarcoidosis
B. Late stage porphyria
C. Contact dermatitis
D. Urticaria
E. Erythema Multiforme

15. A 42 year old patient with systemic sclerosis and extensive sclerosis has been diagnosed with
nephropathy and colorectal cancer. The most likely antibodies present in this patient are:
A. anti-U3 RNP
B. ACA
C. Anti- RNA polymerase III
D. Anti topoisomerase I
E. Anti-U1 RNP

16. Select the true statement for toxic epidermal necrolysis:


A. Can not be caused by the administration of drugs
B. In most cases, the etiologic agent is a mycoplasma infection
C. SCORTEN scale is used for assessing the extent of skin involvement
D. mucosal involvement occurs in the majority of patients
E. average mortality is 1 to 5%

17. Indicate the best association for antibodies in Lupus erythematosus:


A. anti-U1 RNP - drug induced systemic lupus erythematosus
B. anti-Sm - involvement of CNS
C. Anti-Histone antibody - neonatal lupus erythematosus
D. anti-dsDNA - Hypersensitivity to UV
E. Anti-ro - renal involvement
18. Choose from the strongest glucocorticoid for topical use below:
A. betamethasone dipropionate
B. clobetasol propionate
C. flumetasone pivalate
D. hydrocortisone acetate
E. mometasone furoate

19. Cytocellular carcinoma:


A. is a type of actinic keratosis
B. is typically located in non-UV exposed areas
C. is a hyperkeratotic papule
D. can self-smooth (self-heal?)
E. episode is typical in children with AK disorders

20. The Argyll-Robertson symptom may be found in:


A. Chlamydia
B. Syphilis
C. Gonorrhea
D. Herpes
E. In any of the diseases listed above

21. Vaginal discharge with a bad fish-like odor are most likely associated with:
A.Chlamydia trachomatis
B.Neisseria gonorrhoeae
C. Candida albicans
D. Ureaplasma urealyticum
E. Gardnerella vaginalis

22. In the 68-year-old female patient there was distal stenosis of the left lower leg with sclerosis
covering a distance of about 10 cm. and swelling of the proximal part. The most probable cause is:
A. diabetes
B. relapsing baptism
C. concurrent connective tissue disease
D. venous insufficiency
E. monoclonal gammopathy

23. The criterion for the diagnosis of systemic sclerosis (ACR / EULAR 2013) is:
A. calcinosis
B. hypertension
C. lung fibrosis
D. kidney fibrosis
E. narrowing of the labial lip behind the skin

24. Indicate the true statement for squamous cell carcinoma:


A. phototherapy increases the risk of developing
B. typically the border is pearly
C. In the dermoscopic picture there are structures of the D maple leaf type.
D. In the verrucous carcinoma, metastases quickly arise.
E. should be removed surgically with a margin of 1-3 mm of healthy skin

25. The patient with inguinal skin developed acutely blurred patches with slight exfoliation. In the
light of Wood's lamp, an orange-red fluorescence was found. In this case, the proper diagnosis and
treatment are:
A. Erythrasma and erythromycin
B. Tinea of the groin and terbinafine
C. Yeast and fluconazole
D. Psoriasis inversa and topical glucocorticoids
E. Bacterial eczema and fusidic acid

26. Nail mycosis caused by dermatophytes usually is caused by:


A. Trichophyton rubrum
B. Epidermophyton floccosum
C. Trichophyton schoenleinii
D. Trichophyton verrucosum
E. Trichophyton violaceum

27. The patient was diagnosed with erythroderma, which began with lesions located in the interlum
region. Histological examination revealed parakeratosis, atrophy of the granular layer and
lengthening of the skin nodules. The most probable diagnosis is:
A. atopic dermatitis.
B. Norwegian scabies.
C. pemphigus
D. drug-related changes.
E. psoriasis

28. In a patient allergic to Birch allergens, a tingling sensation within the lips and swelling of the oral
mucosa occurs, after eating an apple. The most likely diagnosis is:
A. Hereditary angioedema
B. acute allergic urticaria
C. contact dermatitis (Apple allergens)
D. oral allergy syndrome (OAS)
E. latex syndrome

29. Melanocytic nevi are:

A. nevus flammeus
B. naevus coeruleus
C. nevus sebaceous
D. naevus simplex
E. nevus comedonicus

30. Choose an INCORRECT combination:


A. Lupus TB - incurable scar
B. Sarcoidosis - Deep ulcers
C. Albinism - hashimoto disease
D. Hailey hailey disease - acantholysis
E. Pediculosis Pubis - Pediculus vestimenti

31. Common warts:

A. Only affects the adults (70% in children)


B. can be caused by HPV 2 virus
C. typical localization are the proximal parts of the limbs (distal)
D. primary eruption is a few millimeters of nodes (papules)
E. the legions are accompanied by severe pruritus (pain)

32. The correct statement for venous leg ulcers is:

A. significant pain due to the elevation of the limb


B. Skin around the ulcer is not changed
C. Occurs more often than arterial ulcers
D. Most commonly affects men about 40 years of age
E. typically there are numerous small ulcerations

33. In a 28 year old patient inflammatory nodules have appeared on the extensor surfaces of her
calves, healing without scarring. Which condition can be suspected in the patient.

A. Hypercholesterolemia
B. Atopy
C. Venous insufficiency
D. Diabetes
E. Sarcoidosis

34. An unusual location for scabies is:


A. Around the navel
B. Skin folds
C. Around the interdigital area
D. Around the genitals
E. Interdigital region

35. Anti-PM-Scl are a marker of:


A. Morphea
B. Systemic sclerosis
C. Dermatomyositis
D. Scleromyositis
E. scleromyxedema

36. Indicate the true statement for Raynaud's syndrome:

A. the only cause is low temperature


B. can affect the nose and the ears
C. is the primary symptom of systemic sclerosis
D. sildenafil is a triggering agent
E. treated by glucocorticosteroids

37. In a 60 year old patient, you find round bullae on the skin. Histological examination revealed a
acantholysis just under the stratum corneum and Anti desmoglein 1 antibodies in blood serum. The
most probable diagnosis is:

A. dermatitis herpetiformis
B. pemphigus foliaceus
C. bullous pemphigoid
D. pemphigus herpetiformis
E. linear IGA bullous dermatosis

38. Choose the correct combination of atopic dermatitis and its treatment:

A. treatment with clobetasol


B. Pimecrolimus - chronic treatment should not be used on the eyelid and genitals
C. side effects of tacrolimus: pruritus and burning
D. hydrocortisone - proactive therapy
E. emollients should not be used for less than 3 months

39. Match the correct sentences of the disease with their typical localization:
A. yellow nodular knots - eyelids
B. erysipelas - lower leg
C. SLE - scalp hair skin
D. Necrobiosis lipoidica - trunk
E. Impetigo contagiosa - face

40. In a patient with advanced basal cell carcinoma, in which there are contraindications for using
surgical and radiotherapy treatments, which of the following can be used:
A. Vemurafenib
B. Vismodegib
C. Ipilimumab
D. Nivolumab
E. Trametinib

41.Indicate the true sentence about genital herpes

A. Changes in the genital region are always caused by HSV 2 virus


B. acyclovir therapy should be considered after 6 relapses a year
C. the primary infection does not differ in symptoms from relapses
D. Herpes simplex is accompanied by general symptoms
E. There are no enlargement of lymph nodes

42. indicate the true statement for lupus erythematosus:

A. SLE may be associated with serum albumin


B. DDLE is the variant of SLE
C. SCLE is a form without involvement of internal organs
D. Lupus band test is negative in SLE
E. DLE type changes do not occur in patients with SLE

43. Indicate the correct combination of disease and treatment:

A. Pemphigus vulgaris - etanercept


B. squamous cell carcinoma - Ustekinumab
C. plaque psoriasis - secukinumab
D. idiopathic urticaria - infliximab
E. BCC metastases - ipilimumab

44. What is the correct about Skin tuberculosis


A. Warts are typically found on the skin of the hands
B. Changes in tuberculosis lupus never regress spontaneously
C. Tuberculosis ulcerosa is usually not accompanied by tuberculosis with organ involvement
D. Tuberculosis colliquativa is healing without scarring
E. Tuberculosis lupus displays a negative tuberculin test

45. In a patient with dermatomyositis, we typically find elevated values of:


A. Aldolase and lactate dehydrogenase
B. Creatinine and albumin
C. C3 and C4 constituents
D. Erythrocytes and leukocytes
E. Thrombocytes and concentration of total protein

46. The most common symptom of systemic sclerosis associated with skin sclerosis are:
A. Dysphagia
B. Pulmonary fibrosis
C. Joint pain
D. Raynaud’s syndrome
E. Skin calcinosis

47. Mark the correct sentence


A. Medications are inducing agents
B. The pathologic changes are usually found in the areas without sun exposure
C. In harlequin babies is an autosomal dominant disease in which the lesions appear in the first days of
life
D. Scabies in children is typically found in the skin of the face
E. Bullous impetigo in babies is an acquired disease

48. Choose the correct combination of disease and treatment:

A. acute urticaria - acetylsalicylic acid


B. chronic urticaria - sildenafil
C. chronic idiopathic urticaria - omalizumab
D. Urticaria pigmentosa - acitretin
E. Urticaria due to cold- chloroquine

49. Drugs typically causing Lupus Erythematosus are:


A. Penicillamine, captopril
B. Hydroxychloroquine, chloroquine
C. Penicillin, cephalosporin
D. Hydralazine, procainamide
E. Azathioprine, mycophenolate mofetil

50. Lesions found in HSV infections are the following:


A. ?
B. Is accompanied with vesicles
C. Molluscum contagiosum
D. Vesicular eczema
E. Ecthyma
51. Which of the following is correct:
A. Furuncles do not cause subjective symptoms
B. Furuncles is a follicular inflammation accompanied by streptococcus pyogenes
C. Furunculosis is an inflammation of many neighbouring abscesses
D. Carbuncles are a result of returning furuncles
E. Diabetes increases the amount of furuncle formation

52. Indications for treatment with fractional CO2 laser are:

A. vascular hematoma
B. Pigmented nevi
C. epidermal nevi
D. Herpes simplex infection
E. telangiectasia

53. Buschke-Lowenstein’s large genital warts


A. HPV 6 or 11
B. HPV 13 or 32
C. HSV 2 or 1
D. HSV 6 or 11
E. HPV 33 or 66

54. Glucocorticosteroids are mainly used in:


A. Systemic sclerosis
B. Pustular Psoriasis
C. Persistent Erythema
D. Rosacea
E. Pemphigus Vulgaris

55. In which of the following nevi can we see loss of color


A. Nevus araneus
B. Nevus verrucosus
C. Nevus epidermalis
D. Nevus sutton
E. Nevus spitz

56. In a patient with contact allergy to chromium, the most characteristic is the worsening of eczema
after contact with:

A. Jewelry from non precious metals


B. Rubber
C. Cosmetics
D. Cement
E. Hair paint

57. A 70 year old patient develops a nodular change in the temporal region of the face . In
dermoscopy,branching like vessels were found. In such a case one should first suspect:

A. keratoacanthoma
B. Basal cell carcinoma
C. Squamous cell carcinoma
D. HPV induced SCC
E. Amelanotic melanoma

58. Indicate the correct statement about Mycosis Fungoides:


A. Metophyma are changes in the face in the form of nodules
B. Atypical Granulocytes predominate in histopathology (lymphocytes - Pautrier’s Abscesses)
C. Prognosis is improved by early start of chemotherapy
D. In the Plaque Stage, there are no systemic symptoms
E. A folliculotropic form exists with involvement of the hair follicles
59. Corona phlebectatica paraplantaris
A. Typical localization is in the popliteal area
B. Can indicate a disorder of the deep veins
C. Related is to critical venous insufficiencies
D. Is a complication of Raynaud phenomenon in toes
E. Inherited malformation of veins in the lower extremities

60. Choose the correct statement about alopecia areata


A. Heterogeneity of hair width is a characteristic in alopecia areata
B. Caused by zinc deficiency
C. In dermoscopy we can observe inflammatory changes
D. In AA universalis is characterized by scarring alopecia
E. Associated with auto immunological diseases of the thyroid

61. Retinoid used topically in acne vulgaris is:


A. Benzoyl peroxide
B. Adapalene
C. Ivermectin
D. Azelaic acid
E. mupirocin

62. Mark the correct answer for erythema nodosum and nodular vasculitis
A. Are typically found in men
B. Lesions are non permanent
C. The legs are the most common locations
D. The lesions don't come back
E. The lesions are scarring

63. Indicate the correct combination of disease and treatment:


A. Erythema fixum - sulfones
B. Erythema gyratum repens - hydroxychloroquine
C. Erythema induratum - isotretinoin
D. Erythema nodosum - acitretin
E. Erythema chronicum migrans - Doxycycline

64. Hoigne syndrome (a pseudo-anaphylactic reaction characterized by acute neurological and


psychiatric symptoms) is a side-effect of treatment with:
A. Intravenous crystalline penicillin
B. Intramuscular procaine penicillin
C. Oral phenoxymethylpenicillin
D. Intravenous benzathine benzylpenicillin
E. Intramuscular ceftriaxone

65. Primary lesion in Psoriasis is:


A. Papule or pustule
B. Papule or nodule
C. Nodule or macule
D. Papule or vesicle
E. Papule or macule

66. Nodules are a lesion that appear in:


A. Erythema fixum
B. Erythema dyschromicum perstans
C. Erythema necrolytic migrans
D. Erythema gyratum repens
E. Erythema induratum

67. A 67 year old female patient has progresive loss of hair with the regression of the hair line and
loss of eyebrows. What is the most probable diagnosis
A. Pattern hair loss (androgenetic alopecia)
B. Alopecia areata
C. Cicatracial alopecia
D. Telogen effluvium
E. Anagen effluvium
68. Choose the correct statement about melanoma:
A. Acral lentiginous melanoma appears after trauma
B. In early diagnosis of melanoma we use USG of the skin
C. In case of suspicion of melanoma, an excisional biopsy is performed without healthy skin margin
D. The greater the Breslow Scale, the higher the survival rate of the patient
E. The most common form is superficial spreading melanoma

69. In a 6 year old child, on the scalp there is a patch of alopecia with brittle hair. Under the Wood
lamp the lesion has a green fluorescence. The most probable cause of the disease is:
A. Malassezia (pale yellow)
B. Epidermophyton
C. Candida
D. Trichophyton (none)
E. Microsporum

70. Depigmentation on trunk is the most probably caused by:


A. Erythrasma
B. Pityriasis versicolor
C. Pityriasis seborrhoica
D. Pityriasis rosea Gibert
E. Pityriasis rubra pilaris

71. [can’t read the question]

72. Antiparasitic drug used topically in the treatment of rosacea:


A. Azelaic acid
B. Ivermectin & Metronidazole
C. Brimonidine
D. Benzoyl peroxide
E. Fluconazole
73. Small vessel vasculitis with immune complexes deposition:
A. Microscopic polyangiitis
B. Nodular vasculitis
C. Giant cell arteritis
D. Kawasaki Disease
E. Vasculitis in cryoglobulinemia

74. Lichen planus of the scalp:


A. Does not cause symptoms
B. Causes non-scarring alopecia
C. More common in females
D. Is associated with scarring alopecia of the axillae
E. Most common triggers are infection with gonorrhea

75. Symptom of tabes dorsalis is:


A. Episodic hypertension
B. Impotency
C. Cerebellar ataxia
D. Palsy of the CN IV
E. Hyperreflexia of the tendon and deep reflexes

76. Linear loss of epidermis and dermis is:


A. Erosions
B. Crust
C. Fissure
D. Scar
E. keloid

77. In a 40 year old male, in the region of the beard there are papules, pustules and pustular nodules
that have a tendency to confluence. The most probable diagnosis is:
A. Acne vulgaris
B. LIchen planopilaris
C. Ecthyma
D. Folliculitis
E. Pustular psoriasis

78. In an 8- year old child, on the skin of the neck appear multiple increasing in size lesion
erythematous and scaling patches with vesicles on the borders with pruritis. The most probable
diagnosis is:
A. Erysipelas
B. Tuberculosis LE
C. Erysipeloid
D. Fungal infection
E. Granuloma annulare

79.

80. Parakeratosis is:


A. subungual hyperkeratosis.
B. non-agronomic hyperkeratosis.
C. the presence of cellular nuclei in the stratum corneum.
D. disappearance of stratum corneum.
E. keratosis in the course of skin tumors

81. Major criteria for diagnosis of atopic dermatitis include:


A. Intolerance to wool and food
B. atopy of the patient and atopy of the family history
C. Xerosis and keratosis follicularis
D. food intolerance and white dermographism
E. Onset in childhood and elevated IgE levels in serum

82. A true statement for psoriasis:


A. in general pustular psoriasis, a heliotrope symptom is present.
B. scalp skin psoriasis typically causes alopecia.
C. there is thrombus formation within the nail plates.
D. does not cause pruritus.
E. UV radiation is not one of the provocative factors

83.

84. Indicate the true association of the location and typical clinical manifestation in lichen planus:
A. genital mucosal membrane of women - nephritis
B. nails - the symptoms of an oil stain
C. mucous membrane of the mouth - petechiae
D. hairy skin - pustules and papules coated with a husk
E. smooth skin - scarring

85.
86.

87. Vasculitis with concomitant bronchial asthma and ANCA antibodies is


A. eosinophilic granulomatosis with polyangiitis (Churg and Strauss syndrome)
B. granulomatosis with vasculitis (Wegener's granulomatosis)
C. Takayasu disease
D. giant cell arteritis
E. polyarteritis nodosa

88. In 72-year-old patients with lung cancer erythematous-exfoliative lesions on the ear, ears, hands
and feet were found. The most probable diagnosis is the team of:
A. Gardner
B. Gillin
C. Löfgren
D. Refsum
E. Bazex

89. Give the correct statement:


A. normal warts - the symptom of Auspitz
B. pecherzic Hailey-Hailey - Gottron's symptom.
C. Infectious mollusc - Gardner's symptom.
D. Oral disorder - Asboe-Hansen's symptom.
E. Permanent shadow - Hutchinson's symptom

90. The real statement for SCLE is:


A. characterized by outstanding hypersensitivity to UV
B. In adults are a revelator of organ
C. ANCA D is an immunological marker.
D skin changes disappear with scarring
E. the phisiycosymic variant typically coexists with psoriasis of the 91th.

91. On a 22-year-old patient’s skin of the scalp, nasolabial folds, near the eyebrows and behind the
ear are present erythematous-exfoliative lesions. The most probable diagnosis is:
A. Chronic folliculitis
B. Seborrheic dermatitis
C. Lice
D. Dermatomyositis
E. Rosacea

92. Pre-exposure prophylactic treatment for HIV infection:


A. Is used in HIV positive patients who have multiple risky sexual contacts
B. Is used in pregnant women with HIV infection to prevent the vertical transmission of the virus
C. Is used in patients who do not have HIV but have multiple risky sexual contacts
D. It was proven to be 100% effective in preventing HIV infection
E. Usually is 3 medications are used together

93. Choose which disease frequently occur together:


A. Nail psoriasis and psoriatic arthritis
B. Morphea and systemic sclerosis
C. Erysipelas and erysipeloid
D. Pityriasis rubra pilaris and folliculitis
E. Palmoplantar psoriasis and keratoderma

94. A characteristic of both pemphigus vulgaris and foliaceous is:


A. Intraepidermal acantholysis
B. Granular deposition of IgG and C3 in the basement membrane (pemphigoid)
C. Subepidermal blisters
D. Antibodies against collagen type VII (EBA)
E. Immunoglobulin deposition in papillary dermis

95. Sulfa drugs are a drug of choice in the treatment of urticaria:


A. Associated with intolerance to gluten
B. Vasculitis
C. Cholinergic
D. Mechanical
E. Serum sickness disease

96. In a 56-year old woman, erosions on the oral mucous membrane were found. In histopathological
examination there was acantholysis right under the epidermis. In the serum of this patient we will
find antibodies against:
A. BP180
B. Desmoglein 3
C. Desmoglein 1
D. Topoisomerase 1
E. BP230

97. The risk for the development of skin cancer is not:


A. Immunosuppressive treatment
B. Positive individual interview for skin cancers
C. Professional exposure to UV radiation
D. Local osteonecrosis
E. Retinoid treatment
98. SLE typically do not have antibodies:
A. SES -ANA
B. ds-DNA
C. Sm
D.U1-RNP
E. Ro

99. True statement for rosacea:


A. Primary eruptions are closed comedones
B. Ophthalmic form of doxycycline is used in the treatment
C. UVB 311nm can be used on the erythematous area
D. In the course of the disease there is a disappearance of the sebaceous glands
E. Pustular eruptions are caused by a Staphylococcus aureus infection

100. If the patient is diagnosed with contact allergy to topical glucocorticosteroids, then:
A. change to a less potent form of the drug
B. It is not allowed to use glucocorticosteroid
C. to reduce the use of glucocorticoids to a few days
D. change to a larger formulation
E. change the power of action to a preparation with a different spatial structure

2018 FINAL - POLISH DIVISION

14. Correct statement for DLE:


A. is more common in males
B. a marker of disease is antibodies against RNP
C. lesions are only present on sun exposed areas
D. lesions are located only on hairless skin
E. lesions can cause scarring

15. Precancerous lesions is:


A. acanthosis nigricans
B. keratosis pilaris
C. arsenical keratosis
D. seborrheic keratosis
E. acquired keratoderma of hands and feet

16. In 60 year old patient with erythematous changes and edema on the face and neck with erythema
on the skin over the joins on the hands , presents with weakness of proximal joints. The most
probable diagnosis is:
A. fascitis eosinofilica
B. scleromyxedema
C. lichen sclerosus
D. scleroderma
E. dermatomyositis

17. Furunculosis is:


A. carbuncles
B. furuncle on the upper lip
C. furuncle in children
D. many furuncles in different time of puberty
E. furuncles caused by other bacteria than staphylococcus aureus

[18-40 missing]

41. Acitretin in psoriasis


A. cannot be used in multi drug therapy
B. maximal effect of therapy is observed 2 weeks after starting
C. is a treatment of choice for psoriatic erythroderma
D. causes decrease of cholesterol and triglycerides
E. contraception need to be used for 1 month after the treatment

42. Choose the correct statement about melanoma


A. does not involve mucous membranes
B. skin phototype III and IV are a risk factor
C. previous melanomas are not a risk factor for melanoma
D. many nevi are not a risk factor for the development of melanoma
E. trauma to a pigmented nevus is not a risk factor for the development of melanoma

43. Carcinoma in situ is:


A. Darier disease
B. Hailey- Hailey disease
C. Duhring disease
D. Heck disease
E. Bowen disease

44. Hard nodules with central depression containing granular content localised on the skin of the
fingers and face is most likely:
A. molluscum contagiosum
B. erythema induratum
C. nodular vasculitis
D. common wart
E. flat wart

45. Scarring alopecia appears in:


A. lichen planus and DLE
B. trichophytosis and trichotillomania
C. telogen effluvium and SLE
D. pattern hair loss and alopecia aerate
E. atopic dermatitis and syphilis

46. A female patient experiences inflammation of the lateral nail fold with discharge. The most likely
diagnosis is:
A. lichen planus
B. psoriasis of nails
C. infection with candida albicans
D. nail changes in alopecia areata
E. infection with trichophyton mentagrophytes

[47-64 missing]

64. Choose the correct statement for acne vulgaris


1. the main factor of pathogenesis is aerobic bacteria (anaerobic)
2. a risk factor for increased severity is diet with a high glycemic index
3. noninflammatory lesions are comedones
4. benzoyl peroxide can cause drug resistance
5. treatment of severe acne we can use ivermectin

65. Sweet syndrome is characterised by:


A. lymphopenia with thrombocytopenia
B. leukopenia with eosinophilia
C. thrombocytopenia with lymphopenia
D. leukocytosis with eosinophilia
E. leukocytosis with neutrophilia
66. Acquired ichthyosis is associated with:
A. Hodgkin disease
B. colon cancer
C. Castelman disease
D. prostate cancer
E. lung cancer

67. Choose the correct disease and its treatment:


A. SCC - vemurafenib
B. actinic keratosis - tacrolimus
C. BCC - vismodegib
D. melanoma - Ingenol mebutate
E. HPV induce SCC - omalizumab

68. The most characteristic feature of active alopecia areata in dermoscopy is:
A. “exclamation mark” hair, broken hairs and black dots
B. yellow dots, lanugo hairs
C. hair regrowth, lanugo hairs, ?round hairs
D. cork-screw hairs, broken hairs, hairs in a shape of a comma
E. yellow dots, scales, glomerular vessels

69. Choose the correct association:


A. erythema fixum - gottron sign
B. melanoma - Asboe-Hansen sign
C. erythema nodosum - Nikosky sign
D. scleroderma - Raynaud phenomenon
E. flat warts - Koebner phenomenon

70. In rosacea there are no:


A. telangiectasis
B. comedones
C. nodules
D. papules
E. erythema

71. In a 7 year old child has disseminated yellow-brownish and red-brownish macules on the trunk
and infiltrated nodules with become edematous after rubbing.
A. atopic dermatitis
B. granuloma annulare
C. acanthosis nigricans
D. urticaria pigmentosa
E. familial dysplastic melanocytic nevus syndrome
72. Tuberculin test is negative in tuberculosis:
A. lupus vulgaris
B. tuberculosis verrucosa cutis
C. acute miliary tuberculosis
D. orificial tuberculosis
E. scrofuloderma

73. In a 45 year old patient with diabetes mellitus type II, on the extensor surface of shins, indurated
lesion appear that are yellowish with central clearing. What is the most likely diagnosis:
A. varicose veins
B. erysipelas
C. necrobiosis lipoidica
D. erythema induratum
E. nodular vasculitis

74. Secondary lesions is not:


A. fissure
B. Excoriations
C. scale
D. scar
E. tumor

75. Choose the correct statement about sarcoidosis:


A. If it occurs with erythema nodosum it is called Bazex syndrome
B. nodular changes typically undergo ulcerations
C. vascular form manifest with Raynaud phenomenon and ulcerations of fingers
D. Associated with cystic bone loss
E. diascopy test is negative (apple jelly colored lesions)

76. In a 50 year old patient with alcoholic liver disease and hypersensitivity for UVA, localised vesicles,
erosions, scars and sclerosis of the skin appeared in the place of trauma. What is the most likely
diagnosis and how should it be treated?
A. pemphigus vulgaris and prednisone
B. Darier disease and acitretin
C. morphea and cephalosporins
D. Porphyria cutanea tarda and chloroquine
E. bulls erysipelas and penicillin

77. Choose the correct association between appearance in dermatoscopy and the disease:
A. strawberry-like — BCC
B. black dots — SCC
C. maple leaf structures - BCC
D. branching vessels — actinic keratosis
E. white-yellow scales — BCC

78. Erysipeloid:
A. incubation period is 2 to 7 days
B. the most common localisation is the face
C. is a complication of erysipelas
D. the cause if Actinomyces israeli
E. skin lesions appear with a fever

79. Blue nevus is


A. vascular
B. apocrine
C. epidermal
D. sebaceous
E. melanocytic

80. Choose the correct statement about mycosis fungoides:


A. usually start from tumor stage
B. tumor stage on the face is called rhinophyma
C. typically there is no pruritus
D. for treatment we can use bexarotene
E. causes greater mortality than Sezary syndrome

81. The most common cancer in people with acanthosis nigricans is:
A. nipple cancer
B. lung cancer
C. prostate cancer
D. cervical cancer
E. gastric cancer

82. Neoplastic transformation of parapsoriasis lesion are associated with what clinical manifestation:
A. pruritus
B. burning of the skin and paresthesia
C. formation of pustules
D. hypersensitivity to UV
E. involvement of the face

83. Select the correct associations:


A. Darier disease - hyperkeratotic papules in sebaceous regions
B. Hailey-hailey disease - subepidermal blisters
C. keratosis follicularis - pruritus and burning of the skin
D. tinea capitis (trichophytosis)- scaring
E. tinea capitis (microsopridium) - orange under the Wood lamp

84. Treatment with IVIG and cyclosporine is used in:


A. shingles
B. toxic epidermal necrolysis
C. erysipelas bullosum
D. staphylococcal scalded skin syndrome (SSSS)
E. actinomycosis

85. Topical glucocorticosteroids is a correct treatment in:


A. perioral dermatitis
B. seborrheic dermatitis
C. dermatitis herpetiformis
D. atopic dermatitis
E. SSSS

86. Choose the correct association between monoclonal antibodies and its therapeutic use:
A. adalimumab - IL-17- psoriasis
B. ustekinumab - IL12/23 - SCC
C. rituksimab - CD20 - pemphigus vulgaris
D. omalizumab - IgE - psoriatic arthritis
E. ipilimimab - PD1 - melanoma

87. Correct statement about HPV warts is:


A. Primary lesion is a nodule
B. Plantar warts are usually caused by HPV 13 and 32
C. Common warts are mainly caused by HPV 16 and 18
D. Flat warts are mainly caused by HPV 1 and 2
E. Typical localisation of flat warts is face and dorsum of hands

88. Pautier microabscesses are a common histological finding in:


A. mycosis fungoides
B. psoriasis
C. pyoderma gangrenosum
D. acne vulgaris
E. impetigo contagiosa

89. Intramuscular injection with triamcinolone is used in:


A. nail lichen planus
B. scalp psoriasis
C. systemic scleroderma with involvement of the oesophagus
D. eczema herperticum
E. carbuncle

90. Choose the correct incubation period


A. genital herpes - 4 to 6 weeks
B. syphilis - 5 to 10 days
C. gonorrhoea - 3 to 7 days
D. non-gonorrhoeal urethritis - 7 to 14 days
E. genital warts - 7 to 10 days

91. Choose which of the following pathogen is the cause of female infertility
A. Neisseria gonorrhoea
B. Trichomonas vaginalis
C. chlamydia trachomatis
D. gardnerella vaginalis
E. Mycoplasma ….

92. In a patient with painless ulcer on the head of the penis lasting for over 5 months, it is necessary
to
A. take a swab from the ulcer and examine it under dark field microscopy
B. take a biopsy for histopathological examination
C. perform a culture
D. do serological studies sing VDRL and FTA
E. start antibiotic therapy

93. Which AIDS defining infectious illness allows you to diagnose AIDS in a patient who is HIV+:
A. angiomatosis bacillaris
B. Hairy cell leukaemia
C. eosinophilia
D. Norwegian scabies
E. ulceration of the skin or mucous membrane due to HSV that persists for over 1month

94. In a 30 year old patient, there is a discharge from the urethra, papulonodular lesions on the head
of the penis, nodules on the mucous membranes as well as erythema and edema of the right knee
joint. The most probable diagnosis is:
A. pelvic inflammatory disease
B. disseminated gonorrhoea
C. acute retroviral disease
D. Reiter syndrome
E. Reiter syndrome and disseminated gonorrhoea

95. 61 year old patient before a schedule operation for varicocele veins of the power limb come to the
clinic because of positive TPHA. She has a husband for 30 years, she denies risky sexual contacts with
strangers and other STI. What test do we need to order?
A. USR, TPPA
B. VDRL, TPI
C. VDRL, FTA
D. TPI, FTA
E. IgM EIA, TPI

1. Molluscum contagiosum in HIV-infected individuals is:


a. A large eruption
b. A large eruption located mainly on the face
c.?
d.?
e.?

2. Common to DLE and porphyria cutanea tarda is:


a. keratosis pilaris
b. hepatomegaly
c. The presence of characteristic autoantibodies
D. sensitivity to UVA
E. positive action arachiny

3. Locally we do not use:


a. penicillin
b. mupirocin
c. clindamycin
d.?
e.?
4. In mycosis drobnozarodnikowej (false)
a. Ulam's hair is on the same level
b. spores occur outside and inside the hair
C. fluorescence lamipe Wood
d. can occupy nails
e.?

5. oncogenic types of HPV viruses that


a. HPV 5 and 8
b. HPV 16 18 31 33 66
c. HPV 6 11 42 43 44

6. Scleroderma linearis
a. rarely occurs in children
b. It can lead to distortion and contracture
c. It is preceded by a symptom of Raynaud
d. As part of the states in August przypadadkow antinuclear

7. Characteristic changes in the location of the itch is:


a. Interest in children
b. The sex organs of men
C. abdominal skin
D. interdigital spaces

8. In Lyme borreliosis is not


a. erythema migrans Chronicum
B. Acrodermatitis chronica atrophicans
C. specific IgM in serum acute phase
d. Specific positive reactions kilowych

9. in sarcoidosis is not
a. A positive tuberculin reactions
b. lowering indices odczynowosci cell
c.?
d.?

10. Acute urticaria can be


a. invoked as both an immunological mechanism and nieimmunologicznym
b. lasts no longer than 6 weeks
c. There may be the drugs
D. accompanied by complaints from the digestive tract
e.? This was the incorrect answer

11. antibodies in acute urticaria


a. They are associated with mast cell
b. can be detected in the serum
c. They are associated with basophiles
d. Activate s configuration

12. question about chronic urticaria (something you do not have avoided the anti-leukotriene
drugs)?

13. point of testing (prick-to-prick) and measure the diameter of bubbles and redness in the
places ukltych

14. what relationship evokes immediately after nickel contact dermatitis was most frequently
mentioned epoxy resins chrome cosmetics or detergents, formalin
15. what you need to enter anaphylactic shock - an adrenaline subcutaneously hydrokortyzol
the veins, 300 - 500 mg aminophylline

16. How to treat genital herpes, or orally or topically, and in what doses

17. Criteria (large) diagnosis of AD are:


a. itching
b. Changes in the typical location
c. Personal or family history and atopic
D. chronic and relapsing course and the early beginning
e. Increasing the total concentration of IgE in serum

18. the question of lasting erythema (repent changes exactly in the same places and are the
causative agent of oral medication)

19. psoriasis stawowowa joints which is most often (miedzypaliczkowe hands or feet, and
possibly large joints may be too backbone)

20. Does not occur in psoriasis


a. papillomatosis
b. agranulocytosis
C. parakeratosis
d. hiperkeratosis

21. question about parts of the body occupied by Pityriasis versicolor and pink Gibert

22. androgenic drugs we can use in women in the treatment of acne because they inhibit the
production of sebum and blackheads
23. acne fulminans

24. Jakei proteins are not part of the basilar membrane


a. intermediate filaments
b. lamilaminina
c. collagen IV
d. V collagen
bullous pemphigoid antigen e.

25. dermatitis herpetiformis questions about - IgA antibody macierzypozakomorkowej smooth


muscle, changes in the small intestine - deposits of IgA in dermal papillae

26. question about linear dermatoses Iga

27. in which a symptom of Raynaud disease occurs

28. Jakei factors influence the exacerbation of atopic dermatitis (listed among others were
sweating, too little wigotnosc, stress ..?

29. the question of psoralens

30. the question of basal cell carcinoma


a. has a rippling color and structure obowodze perlowata
b. A variation occurs pigmentosa
c. Has a tendency to bleed
d. very deeply invades podscielisko

31. the question of keratoacanthoma - Histology


32. a few questions about mycosis fungoides% u2013 looks like blood count and T Cell In
relation to what the disease

33. questions about the erythema pelzakowaty nekrolityczny

34. paraneoplastic pemphigus

35. team of Fitz-Hugh-Curtis-response occurs only in women

36. question about baldness in early syphilis-keel small foci

37. changes in dilated capillaries siateczkopodobne% u2013 after antiarrhythmic drugs

38. be notified to the patient with the oil leaking from the coil and epididymitis is directed to a
dermatologist!

39. the band characteristic of dysplastic lesions is% u2026.

40. ulcerative tuberculosis - mouth sores, lack of tuberculin reaction

41. scar in tuberculosis is lupus Unstable because its fire may generate squamous cell
carcinoma

42. Administration of the HPV vaccine will be effective in closing the manifestations of this
vaccine because it has a preventive action

43. what are the factors that cause purpura


44. leg ulcers are associated with chronic venous hypertension because it causes venous trophic
disorders

45. the question of the reconstruction team in AIDS

46. during the asymptomatic patients with HIV are: ...

47. non-specific reaction in syphilis is: ...

48. changes in diabetes

was still a few questions about malignant melanoma, something about perioralis dermatitis,
vasculitis nodosa that deforestation is permanently (areata, bliznowaciejace.) something about
the treatment of gonorrhea.

BOOK QUESTIONS
1. Under wood’s light Microsporum canis fluoresces:
A. Violet
B. Green
C. Yellow
D. Red

2. Under wood’s light Trichophyton violaceum fluoresces:

A. Green
B. Violet
C. Yellow
D. Doesn’t fluoresce

3. Under Wood’s light Malassezia furfur fluoresces:


A. Green
B. Yellow
C. Violet
D. Doesn’t fluoresce

4. Interdigital tinea pedis occurs mostly between the:

A. 4th and 5th toes


B. 2nd and 3rd toes
C. Middle and ring finger
D. All of the above

5. Interdigital Candidiasis occurs mostly between

A. The 4th and 5th toes


B. The 2nd and 3rd toes
C. The middle and ring fingers
D. All of the above

6. Pityriasis Versicolor is caused by

A. Pityrosporum ovale , the pathogenic form of Malassezia furfur


B. Pityrosporum orbiculare the pathogenic form of Malassezia Furfur
C. Malassezia furfur the pathogenic form of Pityrosporum ovale
D. Malassezia Furfur the pathogenic form of pityrosporum orbicular

7. Molluscum contagiosum is caused by :

A. Dna poxvirus
B. Varicella Zoster virus
C. Human papillomavirus

8. Dermatophytes affect all of the following except:


A. Hair
B. Nail
C. Skin
D. Internal organs

9. Herpes progenitalis is an indication for:

A. Normal Vaginal Delivery


B. Termination of pregnancy
C. Cesearian Section

10. Which of the following affects nails:

A. Onychomycosis
B. Psoriasis
C. Lichen Planus
D. All of the above

11. A female patient with bilateral multiple tender nodules on lower limbs can be diagnosed as having

A. Tinea pedis
B. Tinea Circinata
C. Erythema nodosum
D. Erythema Multiforme

12. A 25 Y old patient with well defined erythematous plaques on her face with telangiectasia and
stippling can be diagnosed as having

A. Atopic dermatitis
B. Discoid lupus erythematosus
C. Acne Vulgaris
D. Actinic Lichen Planus

13. A pregnant female with genital warts can be treated with all of the following except :

A. Liquid nitrogen
B. Podophyllin resin
C. Electrocautery
D. Laser

14. Chloroquine is effective in treating:

A. Hypertrophic lichen planus


B. Follicular lichen planus
C. Actinic & mucosal Lichen Planus
D. Vitiligo

15. The most important cause of erythema nodosum is:

A. Streptococcal Infection
B. Sarcoidosis
C. Salmonella
D. Inflammatory Bowel disease

16. Nikolsky sign occurs in:

A. Erythema Nodosum
B. Erythema Multiforme Major
C. Erythema Multiforme minor
D. Erythrasma

17. Causes of physiological alopecias include all of the following except:

A. Post partum
B. Post Irradiation
C. Post Febrile
D. Post operative

18. An infant with oozing erythe on the face and high serum IgE levels s diagnosed as having the
following:

A. Erythrasma
B. Atopic dermatitis
C. PRP
D. Alopecia Areata

19. Ulcerative Impetigo is called:

A. Erythrasma
B. Erysipelas
C. Erythema

20. Urticaria occuring after sweating due to physical exercise is called:

A. Physical urticaria
B. Cholinergic Urticaria
C. Papular Urticaria
D. Pompholyx

21. Erythrasma is caused by

A. Staphylococci
B. Corynebacterium minutissimum
C. B hemolytic streptococci
D. Streptococcus Pyogenes

22. All of the following can appear in EM minor except:

A. Macules
B. Pustules
C. Papules
D. Vesicles
E. Bullae

23. Isomorphic response is present in all of the above except

A. Psoriasis
B. Erythema Multiforme
C. Lichen Planus
D. Plane warts

24. The type of lichen planus which may be complicated by Squamous cell carcinoma is:

A. Hypertrophic type
B. Ulcerative mucosal type
C. Flexural
D. Annular

25. Compulsive plucking of hair is known as:

A. Ophiasis
B. Trichotillomania
C. Alopecia areata
D. Graham little syndrome

26. Erythema? Is present in:

A. Vitiligo
B. Psoriasis
C. Pediculosis
D. None of the above

27. Papular urticaria represents the following types of allergic reactions

A. Type 1
B. Type I and IV
C. Type I and II
D. Type IV

28. Acute Urticaria can be treated by the following

A. Emollients
B. IV steroids
C. Antibiotics
D. AntiSeptics

You might also like